Thursday 15 August 2013

NPTE sample questions with reasoning

hi to my all friends now i posted sample NPTE questions which ill be useful for those who are all preparing for license exams .
1. A physical therapist examining wrist-joint play finds restriction in the
direction indicated by the arrow. The therapist should suspect a decrease 
in which joint motion? 

1. Radial deviation 
2. Ulnar deviation 
3. Flexion 
4. Extension 

Correct Answer: 1
The therapist is shown performing an ulnar glide which is the same joint motion used for radial deviation. Limited motion in this direction indicates limited ability to perform radial deviation.
References:
Kisner C, Colby LA. Therapeutic Exercise: Foundations and Techniques. 4th ed. Philadelphia, PA: FA Davis; 2002: 240.

2. The intervention for a patient with limitation of shoulder flexion and medial (internal) rotation includes mobilization. What glide is MOST appropriate for mobilizing this shoulder to specifically increase the restricted motions? 

1. Posterior 
2. Anterior 
3. Medial 
4. Lateral 

Correct Answer: 1
The most appropriate mobilization technique for increasing both shoulder flexion and medial (internal) rotation would be posterior (dorsal) glide. Lateral glide may be used as a general joint distraction technique. Medial glide would not be appropriate to increase flexion and medial (internal) rotation. Anterior glide is used to increase extension and lateral (external) rotation.
Kaltenborn FM. Manual Mobilization of the Extremities. 4th ed. Oslo, Norway: Olaf Norlis Bokhandel; 1989: 103-114.

3. Which of the following techniques is MOST appropriate for a patient with low postural tone? 

1. Slow regular rocking while sitting on a treatment bolster 
2. Continuous pressure to the skin overlying the back muscles 
3. Low-frequency vibration to the back muscles 
4. Joint approximation applied through the shoulders to the trunk 

Correct Answer: 4
Options 1, 2, and 3 are techniques used to decrease postural tone, which is not indicated for this patient. Option 4 is the most appropriate technique for improving low postural tone.
Voss D, Ionta M, Meyers B. Proprioceptive Neuromuscular Facilitation. 3rd ed. Philadelphia, PA: Harper & Row; 1986: 294, 303-308.
Porter RE. In: Umphred DA, ed. Neurological Rehabilitation. 3rd ed. St. Louis, MO: Mosby; 1995: 549.

4. What skin change associated with aging has the GREATEST effect on wound healing? 

1. Reduction in sensation 
2. Decreased elasticity of the skin 
3. Decreased epidermal proliferation 
4. Change in pigmentation 

Correct Answer: 3 
Wounds heal via a complex process involving re-epithelialization. With advanced aging, the rate of epidermal proliferation decreases.
Kloth LC, McColloch JM, Feedar JA. Wound Healing: Alternatives in Management. 2nd ed. Philadelphia, PA: FA Davis; 1995: 48-49.

5. Which of the following techniques is MOST effective in teaching a patient with insulin-dependent diabetes about foot care? 

1. Reassure the patient that no infections will occur if the directions are followed, then demonstrate procedures. 
2. Tell the patient how foot care is performed, then watch the patient’s performance. 
3. Watch the patient perform a foot inspection and caution him that amputations result from unattended skin problems. 
4. Have the patient demonstrate a foot inspection, then give feedback on the patient’s performance.

Correct Answer: 4
Learning the process of foot care is a psychomotor skill and effective strategies to teach psychomotor skills include repeat demonstration by the patient of the skill followed by feedback from the therapist to highlight what was performed correctly and what areas need improvement, if any. Reassurance about prevention of infection with proper foot care would primarily be a cognitive skill and does not ensure that the patient can effectively perform proper foot care. Options 2 and 3 do not include feedback that informs the patient about their performance.
Shepard KF, Jensen GM. Handbook of Teaching for Physical Therapists. Boston, MA: Butterworth-Heinemann; 1997: 336-337.

6. Utilization review and peer review are activities that are a part of a comprehensive: 
a. policy and procedure manual.
b. quality improvement program.
c. audit cycle.
d. performance evaluation.

Correct Answer: 2
According to the Standards of Practice for Physical Therapy there should be a written plan for continuous improvement of quality care. This includes ongoing review and evaluation of the physical therapy services provided. Utilization review and peer review are two types of review processes.
Standards of Practice for Physical Therapy and the Criteria. In: American Physical Therapy Association. Guide to Physical Therapist Practice. Alexandria, VA: APTA; 1999: Appendix 2.

7. The demographic information of the participants in a research study lists a mean age of 32 and median age of 35. The difference between the median and mean indicates: 
a. the value of the standard deviation score.
b. the value of the Z-score.
c. that the distribution is skewed.
d. that the 2 measures should be averaged.

Correct Answer: 3
A "normally distributed" sample has a median and a mean that are equal in value. In that type of distribution, the median and mean would be at the halfway point. One-half of the scores (50 percent) would be distributed above the median and one-half below. If the median and mean are not of equal value, the distribution is skewed. If the median is of a higher value than the mean, the distribution is skewed to the left, if lower it is skewed to the right. The standard deviation is a measure of the variability of the mean. The Z-score is a standard score with a mean of zero and a standard deviation of one. Averaging the two measures would not be appropriate or meaningful.
Bork CE. Research in Physical Therapy. Philadelphia, PA: JB Lippincott; 1993: 213-219.

8. A physical therapist is treating a young athlete with gastrocnemius muscle strength of Fair plus (3+/5). In the prone position, which of the following exercises is MOST appropriate to maximize strengthening? 
a. Resistive exercises with the knee bent
b. Resistive exercises with the knee straight
c. Active assistive exercises with the knee bent
d. Active assistive exercises with the knee straight

Correct Answer: 2
With a muscle grade of Fair plus, the patient should not need active assistive exercise. Resistive exercise against gravity would be most appropriate to strengthen this muscle. Since the gastrocnemius crosses both the knee and ankle, bending the knee would put the gastrocnemius in a shortened position and lessen its ability to produce tension. Therefore, exercising with the knee straight would put the gastrocnemius on stretch, increasing its ability to produce tension.
Smith LK, Weiss EL, Lehmkuhl LD. Brunnstrom’s Clinical Kinesiology. 5th ed. Philadelphia, PA: FA Davis; 1996: 345-346.

9. Following spinal joint mobilization procedures, a patient calls the therapist and reports a minor dull ache in the treated area of the back that lasted for 2 to 3 hours. Based on this symptom, the therapist should: 
a. consider a possible neurological lesion in the area.
b. refer the patient back to the physician.
c. inform the patient that this response is common.
d. add strengthening exercises to the home program.

Correct Answer: 3
Joint mobilization procedures may cause some soreness. The therapist should inform the patient of this response to treatment. The therapist should re-evaluate the patient and could alter the treatment by waiting an extra day before the next treatment or by decreasing the dosage. There would not be a need to refer to the physician. The addition of exercises would not alter the response and there would be no indication of neurological involvement with the reported symptoms.
Kisner C, Colby LA. Therapeutic Exercise: Foundations and Techniques. 3rd ed. Philadelphia, PA: FA Davis; 1996: 199.

10. A 90 year-old patient with chronic congestive heart failure has been nonambulatory and a nursing home resident for the past year. The patient was recently admitted to the hospital following an episode of dehydration. Which of the following plans for prophylactic respiratory care is MOST appropriate? 
a. Turning, coughing, and deep breathing every 1 to 2 waking hours
b. Vigorous percussion and vibration 4 times/day
c. Gentle vibration with the foot of the bed elevated 1 time/day
d. Segmental postural drainage using standard positions throughout the day

Correct Answer: 1
A patient who is bed-bound and immobile will be prone to developing atelectasis (partial collapse of lung tissue), which can then lead to pneumonia. Frequent position changes with deep breathing and coughing will help prevent development of atelectasis. Given that this patient is elderly and does not have a diagnosis of secretion retention, vigorous percussion and vibration is not indicated. Vibration with the head down or standard postural drainage positions will not be tolerated in this elderly patient with chronic congestive heart failure.
Frownfelter D, Dean E. Principles and Practice of Cardiopulmonary Physical Therapy. 3rd ed. St. Louis, MO: Mosby; 1996: 469-472.

11. To help students apply a newly learned skill to clinical practice, the MOST effective action for the clinical instructor to take is to: 
a. point out possible patient situations and discuss how the skill would apply to them.
b. have the students research reference materials and compile a list of the steps required to acquire the skill.
c. prepare a list of indications and contraindications for the skill.
d. have the students provide examples of patient situations where the skill would be appropriately applied.

Correct Answer: 4
Behavioral objectives should be learner centered, outcome oriented, specific, and measurable. Option 4 is the only one that is learner (student) centered and specific to a situation. Options 1 and 3 require action by the clinical instructor, not the student. Option 2, which may be a step in the process, is not as learner centered or outcome centered as Option 4.
Gandy J. In: Shepard KF, Jensen GM, eds. Handbook of Teaching for Physical Therapists. Newton, MA: Butterworth-Heinemann; 1997: 144-146.

12. To prevent contractures in a patient with a transfemoral amputation, emphasis should be placed on designing a positioning program that maintains range of motion in hip: 
a. flexion and abduction.
b. extension and adduction.
c. adduction and lateral (external) rotation.
d. flexion and medial (internal) rotation.

Correct Answer: 2
Following an above knee (transfemoral) amputation, the residual limb has a tendency to develop contractures in the hip flexors and abductors. Therefore, it is particularly important that the patient be positioned so as to maintain full range of motion in hip extension and adduction.
May BJ. Amputations and Prosthetics: A Case Study Approach. Philadelphia, PA: FA Davis; 1996: 86.

13. After a long-term history of bilateral lower extremity vascular insufficiency, an otherwise healthy patient had a right transfemoral amputation. For this patient, which of the following factors is MOST important in establishing long-term goals for functional walking? 
a. Status of the wound at the amputation site
b. Range of motion of the right hip
c. Condition of the left lower extremity
d. Ability to maintain upright posture

Correct Answer: 3
The left limb must function as the main support limb. Any treatment strategy for ambulation must ensure that the remaining limb is optimally functioning and that the limb is healthy. While the other factors are relevant, the integrity of the remaining limb is the greatest concern for this patient in establishing long term, functional goals.
Mensch G, Ellis P. Physical Therapy Management of Lower Extremity Amputations. Rockville, MD: Aspen; 1986: 107.

14. A physical therapist is conducting a 12-minute walk test with a patient who has chronic obstructive pulmonary disease and uses 2 L/min of oxygen by nasal cannula. The patient’s resting oxygen saturation is 91% and resting heart rate, 110 bpm. The oxygen flow should be increased if the: 
a. patient’s carbon dioxide level starts to increase.
b. patient starts to report shortness of breath.
c. patient’s oxygen saturation falls below 87%.
d. patient’s heart rate is greater than 150 bpm.

Correct Answer: 3
A fall in oxygen saturation below 87 % is equivalent to a partial pressure of 55 mm Hg of oxygen in the blood, which is considered to be moderately hypoxemic (low oxygen levels). This situation would require increased oxygen levels in order to be rectified. A rise is carbon dioxide level would not be alleviated by increased oxygen levels. Complaints of shortness of breath can come from a variety of causes and would not necessarily be alleviated by increased oxygen levels. An increase in HR to 150 bpm may be a normal response to this activity and would not necessarily require increased oxygen levels.
Panik M, Paz JC. In: Paz JC, Panik M, eds. Acute Care Handbook for Physical Therapists. Boston, MA: Butterworth-Heinemann; 1997: 124-125.

15. When training a patient to increase muscle activity with the use of electromyographic biofeedback, the physical therapist should adjust the unit so that sensitivity: 
a. starts low and increases as the patient shows an increase in muscle activity.
b. starts high and decreases as the patient shows an increase in muscle activity.
c. remains at approximately midrange during the entire treatment period.
d. is not set, since this is not necessary for this form of biofeedback.

Correct Answer: 2
Increasing the sensitivity makes the biofeedback unit more sensitive to electrical potentials from muscles. As the sensitivity is decreased, it takes more electrical activity to trigger the biofeedback unit (i.e., provide an audio or visual cue to the patient). For use in muscle re-education, the unit should be most sensitive during the initial treatment so the patient is able to recruit enough motor units to trigger the unit. As the patient is able to recruit more motor units, the sensitivity is decreased which would require the patient to activate more motor units.
Wolf SL. In: Nelson RM, Currier DP, eds. Clinical Electrotherapy. 2nd ed. Norwalk, CT: Appleton and Lange; 1991: 367.

16. During a posture examination, the physical therapist notes that both of the patient’s patella point inward when viewed from the front of the patient. The MOST likely cause of this problem is excessive: 
a. femoral anteversion.
b. weakness of the vastus medialis.
c. genu varum.
d. medial tibial torsion.

Correct Answer: 1
The most common cause of inwardly pointing or "squinting patellae" is excessive femoral anteversion. Although there is normally 8° to 15° of femoral anteversion, an excessive amount leads to squinting patellae and toeing in. The other options would all have a tendency to cause the patella to point outward during standing.
Magee DJ. Orthopedic Physical Assessment. 3rd ed. Philadelphia, PA: WB Saunders; 1997: 475, 514.

17. A physical therapist is evaluating a patient who has a vascular lesion in the brainstem affecting the oculomotor nerve (III). During the cranial nerve examination, which of the following signs would be the MOST significant? 
a. Inability to close the eyelid
b. Medial strabismus
c. Ptosis of the eyelid
d. Constricted pupil

Correct Answer: 3
The oculomotor nerve innervates the levator palpebrae superioris muscle that elevates the upper eyelid and the pupillary constrictor muscle. Therefore, a lesion of the oculomotor nerve would make it difficult, if not impossible, for the patient to fully raise the lid (open the eye) and would cause a condition termed ptosis. In addition, the eye may not react to light and therefore would not show pupillary constriction when light is directed into the eye. Inability to fully close the eye would be seen with a lesion of the facial nerve (Bell’s palsy). Medial strabismus would be caused by damage to the abducens nerve, innervating the lateral rectus causing a medial strabismus.
References: Lundy-Ekman L. Neuroscience: Fundamentals for Rehabilitation. Philadelphia, PA: WB Saunders; 1998: 267-273.

18. A patient is referred to physical therapy for treatment of tenosynovitis. The patient reports a "pins and needles" sensation on the palmar surface of the thumb (1st digit), index (2nd digit), and middle (3rd digit) fingers. The physical therapist’s examination reveals a positive Tinel’s sign at the wrist and Good (4/5) grade opposition of the thumb (1st digit). Based on these findings, the therapist should suspect: 

1. median nerve compression at the wrist. 
2. ulnar nerve compression distal to the elbow. 
3. tenosynovitis of the abductor pollicis longus. 
4. thoracic outlet syndrome. 

Correct Answer: 1 
The median nerve supplies sensory innervation to the palmar surface of the thumb, index and middle fingers. A positive Tinel’s sign (eliciting a paresthesia while tapping over the carpal tunnel at the wrist) and weakness of the opponens pollicis muscle are indicative of carpal tunnel syndrome. Ulnar nerve compression would cause sensory and motor changes in the little and ring fingers not the thumb. Tenosynovitis of the abductor pollicis longus muscle would most likely reveal a positive Finkelstein’s test (stretching of the abductor muscle) with pain over the dorsum of the thumb. Thoracic outlet syndrome would most likely be revealed with special tests that cause alteration of the radial pulse.
Magee DJ. Orthopedic Physical Assessment. 3rd ed. Philadelphia, PA: WB Saunders; 1997: 314-315.

19. A patient with a complete thoracic spinal cord injury is sitting in a wheelchair on a custom made cushion. Pressure relief activities should be performed: 

1. when the patient shows signs of pressure sores. 
2. every 15 to 20 minutes. 
3. every 1 to 2 hours. 
4. if the patient does not have an appropriate cushion. 

Correct Answer: 2 
A patient with a thoracic spinal cord level injury is able to perform independent pressure relief strategies and should be completed every 15 to 20 minutes.
Somers M. Spinal Cord Injury: Functional Rehabilitation. Norwalk, CT: Appleton & Lange; 1992: 92.

20. A therapist is measuring passive knee range of motion in a patient. The measurements obtained are shown in photographs A and B. The MOST likely cause of the difference in knee range of motion is: 

1. knee joint capsule restriction. 
2. tightness in the rectus femoris. 
3. weakness of the hamstrings. 
4. tightness in the vastus medialis . 

Correct Answer: 2 
Capsular restriction would show up in both measurements. In photograph A, there is more knee flexion present with the hip flexed. In this position the rectus femoris is on slack across the hip joint allowing greater range of knee flexion. In photograph B the rectus femoris is stretched over both the knee joint and the hip joint, so tightness in the rectus femoris would restrict knee flexion. Photograph B also shows hip joint flexion. Hamstring weakness would not affect passive range of motion. Vastus medialis tightness would affect both measurements.
Norkin CC, White DJ. Measurement of Joint Motion: A Guide to Goniometry. 3rd ed. Philadelphia, PA: FA Davis; 2003: 232.

21. The hospital administrator asks members of the rehabilitation department to develop a comprehensive program to help reduce the risk of low back injuries. Which of the following steps is the FIRST step necessary to develop this program? 

1. Include all employees in a lumbar extension exercise class. 
2. Design a program that meets each department’s functional needs. 
3. Perform an ergonomic analysis on each workstation. 
4. Provide pamphlets on proper body mechanics. 

Correct Answer: 3 
The first step in preparing any education experience is to determine what the needs of the audience are. Since prevention of low back injury is the ultimate goal, assessment of the employee’s workstations is an important first step in planning the educational program. Simply instructing all employees in lumbar extension exercises is inappropriate without a thorough evaluation of their physical needs. Option 2 cannot be done unless an assessment of the functional needs has been performed. Providing pamphlets are a useful teaching adjunct but would not be the first step to prevent low back pain effectively in this population.
Nemshick MT. In: Shepard KF, Jensen GM, eds. Handbook of Teaching for Physical Therapists. Newton, MA: Butterworth-Heinemann; 1997: 305-310.

22. The physical therapist is positioning a patient for postural drainage. To BEST drain the posterior segment of both lower lobes, the patient should be placed in which of the following positions? 

1. Prone, head down at a 45° angle 
2. Supine, flat surface 
3. Sidelying, head elevated at a 30° angle 
4. Sitting, leaning forward 

Correct Answer: 1 
The best position for the patient, to drain the posterior segment of both lower lobes would be prone lying with the head down and the lower extremities and hips elevated to about 45°.
Panik M. In: Paz JC, Panik M, eds. Acute Care Handbook for Physical Therapists. Boston, MA: Butterworth-Heinemann; 1997: 648.

23. A patient who has a right piriformis syndrome is referred to physical therapy for evaluation and intervention. The patient’s history includes a total hip arthroplasty on the right side 2 years ago. Because of the total hip arthroplasty, which of the following interventions require added precautions for this patient? 

1. Transcutaneous electrical nerve stimulation 
2. Continuous ultrasound 
3. Hot packs 
4. Massage to the right hip 

Correct Answer: 2 
The only one of the above interventions that requires precaution because of the total hip replacement is continuous ultrasound. However, that does not mean that ultrasound is contraindicated for this patient. Transcutaneous electrical nerve stimulation may be used over metal implants. Hot packs and massage would not affect the total hip prosthesis.
Sweitzer RW. In: Hecox B, Mehreteab TA, Weisberg J, eds. Physical Agents. Englewood Cliffs, NJ: Prentice Hall; 1994: 176-178.

24. A patient presents with adhesive capsulitis of the shoulder joint. The range of motion examination reveals restricted lateral (external) rotation and abduction of the shoulder. The FIRST mobilization procedure that should be done for this patient is: 

1. posterior glide. 
2. distraction. 
3. anterior glide. 
4. lateral (external) rotation. 

Correct Answer: 2 
For this patient, the first mobilization procedure would be distraction of the glenohumeral joint. The distraction separates the joint surfaces and is used as a test of joint play. The distraction can also help increase joint play. Distraction may also be used in conjunction with the other mobilization techniques listed. Later mobilization techniques would most likely include anterior glide.
Kaltenborn FM. Manual Mobilization of the Extremities. 4th ed. Oslo, Norway: Olaf Norlis Bokhandel; 1989: 103-114.

25. A patient sustained a severe, traumatic brain injury 3 months ago. During the examination of passive range of motion, the physical therapist notices decreased passive knee extension and moderate hamstring spasticity. Palpation of the knee reveals a firm mass on the lateral aspect of the joint. The therapist should refer the patient to a physician for an evaluation of probable: 

1. osteogenic sarcoma. 
2. patellar fracture. 
3. osteomyelitis. 
4. heterotopic ossification. 

Correct Answer: 4 
Heterotopic ossification occurs in 10 percent to 20 percent of patients with traumatic brain injuries. The abnormal bone formation occurs in the soft tissue surrounding major joints. Patients with spasticity are at increased risk. Loss of range of motion is characteristic for this problem. Osteogenic sarcoma is less likely. A patellar fracture would not present as a lateral mass.
Fredericks CM, Saladin LK. Pathophysiology of the Motor Systems: Principles and Clinical Manifestations. Philadelphia, PA: FA Davis, 1996: 545-547.

26. A patient reports anterolateral shoulder pain with an insidious onset. Examination shows full passive range of motion pain on passive lateral (external) rotation and pain on resistive medial (internal) rotation. These signs are consistent with a diagnosis of: 

1. bicipital tendonitis. 
2. supraspinatus tendonitis. 
3. subscapularis tendonitis. 
4. infraspinatus tendonitis. 

Correct Answer: 3 
Pain with resisted medial (internal) rotation and pain with passive lateral (external) rotation is indicative of subscapularis tendonitis. Bicipital tendonitis is suspected if resisted supination is painful when the patient’s arm is at the side and the elbow is flexed to 90°. Painful resisted abduction and resisted lateral (external) rotation is indicative of supraspinatus tendinitis. Pain on resisted lateral (external) rotation is indicative of infraspinatus tendonitis.
Cyriax J. Illustrated Manual of Orthopaedic Medicine. London, England: OM Publications; 1983: 225-227.

27. During manual muscle testing of the hip flexors in the sitting position, a patient exhibits lateral (external) rotation with abduction of the thigh as resistance is applied. The physical therapist should suspect muscle substitution by the: 

1. sartorius. 
2. tensor fascia latae. 
3. adductor longus. 
4. semimembranosus. 

Correct Answer: 1 
The sartorius flexes, externally rotates and abducts the hip joint. With resisted hip flexion, the sartorius will be recruited to perform all three actions giving the observed substitution pattern. The tensor fascia latae is a medial (internal) rotator and flexor of the hip, so substitution by it would involve medial (internal) rotation and abduction. The adductor longus would adduct the hip. Substitution by the semimembranosus would cause hip extension.
Hislop HJ, Montgomery J. Daniels and Worthingham’s Muscle Testing: Techniques of Manual Examination. 6th ed. Philadelphia, PA: WB Saunders; 1995: 172

28. A patient is entering a cardiac rehabilitation program. The physical therapist should FIRST ask the patient to: 

1. describe the correct aspects of exercise demonstrated by the therapist. 
2. list problems associated with poor nutritional habits. 
3. identify the harmful effects of smoking with regards to cardiac disease. 
4. describe the type of angina that the patient experiences. 

Correct Answer: 4 
In order to best intervene with a patient who has had cardiac dysfunction, a full examination and evaluation is necessary to properly form a treatment plan. An important aspect of the examination is ascertaining the type of angina that the patient experiences so that the therapist will know how to prevent angina with exercise or recognize it if it does occur during the treatment session. The other options provided are all outcomes that would occur after the patient has completed a cardiac rehabilitation program.
Irwin S, Tecklin JS. Cardiopulmonary Physical Therapy. 3rd ed. St. Louis, MO: Mosby; 1995: 113.

29. A physical therapist is setting up a home program of electrical stimulation for a patient who has Bell’s palsy. Which of the following muscles should be stimulated as part of the home program? 

1. Sternocleidomastoid 
2. Masseter 
3. Temporalis 
4. Frontalis 

Correct Answer: 4 
Bell’s palsy involves the facial nerve. The frontalis is the only muscle listed that is innervated by the facial nerve. The sternocleidomastoid is innervated by the spinal accessory nerve, and the masseter and temporalis are innervated by the trigeminal nerve.
Kendall FP, McCreary EK, Provance PG. Muscles: Testing and Function. 4th ed. Baltimore, MD: Williams & Wilkins; 1993: 300-301.

30. A patient is lying supine with hips and knees extended and hands behind the head. The patient is able to raise the head, shoulders, and thorax from the treatment table, but is unable to come to a complete long-sitting position. What muscle should the physical therapist target for a strengthening program? 

1. Iliopsoas 
2. External abdominal oblique 
3. Quadratus lumborum 
4. Upper rectus abdominus 

Correct Answer: 1 
The abdominal muscles are active during a sit-up (with the knees extended) up until the spine is completely flexed (head, shoulders, thorax lifted from surface). In order to come to a long-sitting position however, the hips must be flexed and the abdominals cannot perform this action because they do not cross the hip joint. Therefore, the hip flexors (iliopsoas among others) would have to complete this motion. The inability to achieve a long-sitting position would suggest weakness in the iliopsoas muscle.
Kendall FP, McCreary EK, Provance PG. Muscles: Testing and Function. 4th ed. Baltimore, MD: Williams & Wilkins; 1993: 172-174.

31. A 14 month-old child with spastic diplegia is up on the tiptoes with the toes curled when held in supported standing. This position is characteristic of a: 

1. proprioceptive placing reaction. 
2. moro reflex. 
3. plantar grasp reflex. 
4. traction response. 

Correct Answer: 3 
The plantar grasp reflex is characterized by curling of the toes when a child is held supported in standing. The reflex is normal up to 9 months of age. Delayed integration of this reflex can result in delayed, independent ambulation.
Crutchfield CA, Barnes MR. Motor Control and Motor Learning in Rehabilitation. Atlanta, GA: Stokesville Publishing Co.; 1993: 172.

32. Following trauma at the C5 spinal cord level, a patient was admitted to the hospital. Twenty-four hours later, the patient shows no reflexes, sensation, or voluntary motor activity below the level of injury. These findings indicate: 

1. the presence of spasticity. 
2. decerebrate rigidity. 
3. spinal shock. 
4. a lower motor neuron lesion. 

Correct Answer: 3 
Spinal shock occurs as a reaction to spinal cord injury and is characterized by an absence of all reflex activity below the level of the lesion. Depending on the extent of the lesion, the patient may lose all or some of their sensation and motor activity below the level of the lesion. Spasticity is associated with hyperreflexia and increased muscle tone. Spasticity would be expected to develop following the spinal shock stage. Decerebrate rigidity involves a sustained contraction of the upper and lower extremities in extension. With a lower motor neuron lesion, the loss of sensation and motor activity would be confined to a much smaller region (depending on the exact lesion) and would not affect all levels below the lesion.
Schmitz TJ. In: O’Sullivan SB, Schmitz TJ, eds. Physical Rehabilitation: Assessment and Treatment. 3rd ed. Philadelphia, PA: FA Davis; 1994: 538.

33. A patient with a right transfemoral prosthesis will be able to maintain the knee in extension while weight bearing if the center of gravity of the body is shifted so that the gravitational line falls: 

1. posterior to the axis of the right knee joint. 
2. lateral to the axis of the right knee joint. 
3. anterior to the axis of the right knee joint. 
4. medial to the axis of the right knee joint. 

Correct Answer: 3 
Static alignment for knee stability is established by positioning of the knee so that the lateral reference line falls anterior to the knee joint.
Mensch G, Ellis P. Physical Therapy Management of Lower Extremity Amputations. Rockville, MD: Aspen; 1986: 233.

34. An initial physical therapy evaluation is performed on an elderly patient who is 1 day post total left hip arthroplasty (noncemented) using a posterior-lateral approach. The patient has no complicating medical history and was active and independent preoperatively. Which of the following activities is NOT an appropriate goal for the first week of therapy? 

1. Active-assistive positioning of the left hip to 60° of flexion 
2. Active, left hip abduction in right sidelying 
3. Independent bed mobility with use of a trapeze 
4. Walking with moderate assistance with a standard walker to 25 ft (7.6 m) 

Correct Answer: 2 
Although protocols depend on the surgeon and the approach, it is generally recommended that anti-gravity hip abduction exercises not begin until 5 to 6 weeks post surgery. Patients are taught to avoid excessive hip flexion, usually beyond 80°. Bed mobility and ambulation would be started 1 or 2 days post operatively.
Cameron HU, Brotzman SB, Boolos M. In: Brotzman SB, ed. Clinical Orthopaedic Rehabilitation. St. Louis, MO: Mosby; 1996: 285-294.

35. A patient with frequent tension headaches has been referred to physical therapy for instruction in a program of progressive relaxation exercises. Which of the following is MOST essential in a program of progressive relaxation to reduce muscle tension? 

1. Release of tension by suggestion and persuasion 
2. Passive exercise in quiet surroundings to relieve tension 
3. General massage using deep stroking and kneading of tense muscles 
4. Recognition of the sensations of tension and release 

Correct Answer: 4 
Posterior muscle tension is implicated in the development of tension headaches. Positive imagery is recommended as a self-treatment technique, but the awareness of tension and the appreciation of its absence are foundational. Massage and passive exercise may be relaxing, but the patient must be aware of the sensations of tension to prolong the benefit.
Donkin SW. Sitting on the Job. Boston, MA: Houghton Mifflin; 1989: 74-77.

Goodman CC, Boissonnault WG. Pathology: Implications for the Physical Therapist. Philadelphia, PA: WB Saunders; 1998: 796-797.

36. A patient slips, falls, and cuts her arm in the clinic. The cut is bleeding and the patient is alert and well oriented. In performing first aid for the patient, the FIRST action that the physical therapist should take is to: 

1. don a pair of gloves. 
2. clean the cut with an antiseptic. 
3. check the patient’s blood pressure. 
4. cover the cut with a sterile dressing. 

Correct Answer: 1 
Infection control requires that the wound not be contaminated further, and that the health care workers protect themselves from disease by avoiding contact with body fluids. In this case the patient does not appear to be in life threatening danger, and so the wound should be attended to. Therefore, checking blood pressure would not be the first thing to do. Once the gloves are donned, cleaning the wound and covering it with a sterile dressing would be appropriate.
Rothstein JM, Roy SH, Wolf SL. The Rehabilitation Specialist’s Handbook. 2nd ed. Philadelphia, PA: FA Davis; 1998: 1085.

37. A physical therapist is working with a patient who is aware of being terminally ill. What is the MOST appropriate intervention when the patient wants to talk about the prognosis? 

1. Discourage discussion of death or dying. 
2. Refer the patient for pastoral counseling. 
3. Relate the therapist’s experiences with other patients. 
4. Encourage the patient’s expression of feelings. 

Correct Answer: 4 
Patients should be encouraged to express their feelings. Comparisons to other patients who are dying, in an effort to assure the patient he is not alone, takes away from this patient’s feelings. Denial of death would not be good for the patient, since he must ultimately cope with the inevitable. Pastoral counseling would be an option, but the therapist should be ready to listen to the patient, encourage expression of feelings and avoid denial.
Purtilo R. Health Professional Patient Interaction. 4th ed. Philadelphia, PA: WB Saunders; 1990: 293-296.

38. A physical therapist examines a patient who reports foot pain while jogging. The examination shows that the patient has excessive foot pronation and forefoot varus. The therapist decides to try a temporary orthotic insert in the patient’s running shoe. Which of the following is the MOST appropriate orthotic insert? 

1. A lateral forefoot post under the 5th metatarsal head 
2. A lateral rearfoot post under the calcaneus placed in an everted position 
3. A wedge placed under the instep of the medial foot just beneath the head of the talus 
4. A medial post just proximal to the 1st metatarsal head 

Correct Answer: 4 
Pronation of the foot can be caused by a variety of factors including calcaneal eversion and forefoot varus. Correction of the pronation by an orthosis could include a medial post (wedge) placed just proximal to the metatarsal heads or a medial post under the calcaneus. This approach involves bringing the ground up to meet the foot. A post under the fifth metatarsal head would accentuate the problem, as would a rearfoot post placing the calcaneus in an everted position. If the patient has excessive forefoot varus, a wedge may be placed in the instep in addition to the medial wedge proximal to the metatarsal heads to distribute the load; however a wedge in the instep by itself would not be the best intervention.
Donatelli R, Wooden M. In: Donatelli R, Wooden M, eds. The Biomechanics of the Foot and Ankle. Philadelphia, PA: FA Davis; 1990: 194-196.Michaud TC. Foot Orthoses and Other Forms of Conservative Foot Care. Baltimore, MD: Williams & Wilkins; 1993: 76-77.

39. Although knee motion occurs primarily in 1 plane, tibial rotation is possible when the knee is positioned in 90° or more of flexion because in this position the: 

1. condyles of the femur glide posteriorly on the condyles of the tibia. 
2. hamstrings act as a rotating force. 
3. patella deviates inferiorly. 
4. tension on the ligaments is decreased. 

Correct Answer: 4 
When the knee is extended the medial and lateral collateral ligaments are taut. During knee flexion the ligaments slacken. Therefore there is very little tibial rotation when the knee is extended (closed packed position) and approximately 40° of axial rotation with the knee flexed. Although the femoral condyles may glide posteriorly (depending on the direction of rotation) on the tibia and the hamstrings may rotate the tibia, the reason the motion is available is due to laxity in the collateral ligaments. Other ligaments such as the cruciates and the joint capsule may add to the stability in the closed packed position.
Smith LK, Weiss EL, Lehmkuhl LD. Brunnstrom’s Clinical Kinesiology. 5th ed. Philadelphia, PA: FA Davis; 1996: 305-306.

40. A postural correction program for a patient with forward head, kyphosis, and increased lumbar lordosis should include all of the following EXCEPT: 

1. strengthening the scapular protractors. 
2. strengthening the thoracic erector spinae muscles. 
3. lengthening the short suboccipital muscles. 
4. lengthening the lumbar erector spinae muscles. 

Correct Answer: 1
With this particular posture, the patient’s scapula would be in a protracted (abducted) position; therefore the scapular protractors are already overactive and would require stretching not strengthening. The kyphosis suggests that the thoracic erector spinae muscles are weak and need strengthening. The lumbar lordosis indicates shortened lumbar erector spinae muscles. The forward head posture suggests that the cervical spine is flexed and the occiput is extended, therefore stretching of the suboccipital muscles would be indicated. A TEST-TAKING HINT: Although EXCEPT questions are rarely used on the NPTE, be alert for this type of question and read the responses so that you select the unrelated response.
Kisner C, Colby LA. Therapeutic Exercise: Foundations and Techniques. 3rd ed. Philadelphia, PA: FA Davis; 1996: 534-539. 

41. A patient in the eighth month of pregnancy presents with numbness and tingling of the left hand, except for the little finger (5th digit). She demonstrates edema of the hand and fingers, a positive Tinel’s sign at the wrist, and a Good (4/5) muscle test grade of the wrist and finger flexors. The MOST appropriate intervention is: 

1. a wrist splint to position the wrist in full extension. 
2. a hot pack followed by tendon gliding exercises. 
3. resistive exercises for the wrist and finger flexors. 
4. frequent rest and elevation of the left upper extremity. 

Correct Answer: 4 
Compression on the median nerve (carpal tunnel syndrome) is occurring, most likely as a result of swelling associated with the individual being in the eight month of pregnancy. In this case, rest and elevation would do the most to decrease the edema and relieve the symptoms. The wrist should not be positioned in full extension. Initial conservative treatment sometimes includes cock-up splinting to hold the wrist in neutral to 10° of extension, but not full extension. Although tendon gliding exercises may be used, heat would not be indicated since it may increase the edema. Resistive exercises for the wrist and fingers may aggravate the compression in the carpal tunnel.
Saidoff DC, McDonough AL. Critical Pathways in Therapeutic Intervention. St. Louis, MO: Mosby; 1997: 23-37.

42. A patient who has chronic obstructive pulmonary disease is being treated with a regimen that includes pursed-lipped breathing exercises. The PRIMARY purpose of the pursed-lipped breathing is to: 

1. help prevent the collapse of pulmonary airways during exhalation thereby reducing air trapping. 
2. decrease the removal of carbon dioxide during ventilation. 
3. increase the residual volume of respiration so that more oxygen is available for body metabolism. 
4. stimulate further mobilization of mucous secretions to higher air passages where they can be expectorated. 

Correct Answer: 1 
A patient with COPD has premature collapse of the airways upon exhalation, which leads to air trapping and ultimately poor gas exchange. Breathing out through pursed-lips slows the airflow and creates a back pressure which helps to prevent the airways from collapsing while exhaling. By exhaling more fully through pursed-lips, more carbon dioxide is removed. By preventing airway collapse and air trapping in the lungs, the residual volume is actually decreased. Pursed-lipped breathing helps with ventilation, but does not necessarily assist with secretion mobilization.
Frownfelter D, Dean E. Principles and Practice of Cardiopulmonary Physical Therapy. 3rd ed. St. Louis, MO: Mosby; 1996: 420.

43. A patient with a diagnosis of cervical radiculopathy reports numbness of the right little finger (5th digit). The physical therapist will MOST likely find a diminished tendon reflex in the: 

1. biceps brachii. 
2. deltoid. 
3. triceps brachii. 
4. brachioradialis. 

Correct Answer: 3 
The dermatome providing sensation to the little finger is innervated by the C8 nerve root. The triceps brachii is the only one of the muscles listed that is also innervated by the C8 nerve root. The biceps brachii, deltoid and brachioradialis are innervated by the C5, C6, and sometimes C7 nerve roots.
Magee DJ. Orthopedic Physical Assessment. 3rd ed. Philadelphia, PA: WB Saunders; 1997: 135, 193.

44. In a suction-socket prosthesis, the PRIMARY function of the valve in the lower and medial part of the socket is to permit air to: 

1. remain during the stance phase of gait. 
2. remain during the swing phase of gait. 
3. escape during the swing phase of gait. 
4. escape during the stance phase of gait. 

Correct Answer: 4 
Air is released during the stance phase of gait, which results in a negative pressure inside the socket to provide a suction suspension during the swing phase.
Karacoloff LA, Hammersley CS, Schneider FJ. Lower Extremity Amputation. 2nd ed. Gaithersburg, MD: Aspen; 1992: 46-47.

45. A patient with chronic venous insufficiency of the lower extremities is MOST likely to exhibit: 

1. normal superficial veins, no edema, ulceration, and patches of gangrene around the toes. 
2. dilation of superficial veins, edema, and stasis ulceration. 
3. no edema, cold, hairless extremities, and faint dorsalis pedis pulse. 
4. dilation of superficial veins and edema made worse during sitting or elevation of the lower extremities. 

Correct Answer: 2 
With venous insufficiency, the limbs would be edematous, the superficial veins would be dilated and if not corrected ulceration could develop. Options 1 and 3 are ruled out because they indicate no edema. Option 4 is not correct because the condition is relieved by sitting or leg elevation.
Beers MH, Berkow R. The Merck Manual. 17th ed. Whitehouse Station, NJ: Merck Research Laboratories; 1999: 1793.

46. Following removal of a long-leg cast, a patient has limited knee flexion. The MOST appropriate direction of patellar mobilization is: 

1. distal. 
2. lateral. 
3. proximal. 
4. medial. 

Correct Answer: 1 
In order to improve knee flexion, the physical therapist needs to address patella tightness. During flexion at the knee, the posterior motion of the tibia causes the ligamentum patellae to pull the patella distally and posteriorly. Patellar mobilization in the distal direction would assist with increasing knee flexion.
Norkin, CC, Levangie PK. Joint Structure and Function. Philadelphia, PA: FA Davis; 1992: 366.

47. Which lower extremity proprioceptive neuromuscular facilitation pattern is MOST appropriate for a patient who needs strengthening of the tibialis posterior? 

1. Hip extension, abduction, and medial (internal) rotation, with ankle plantarflexion and eversion 
2. Hip flexion, adduction, and lateral (external) rotation, with ankle dorsiflexion and inversion 
3. Hip extension, adduction, and lateral (external) rotation, with ankle plantarflexion and inversion 
4. Hip flexion, abduction, and medial (internal) rotation, with ankle dorsiflexion and eversion 

Correct Answer: 3 
The tibialis posterior plantar flexes and inverts the foot. This pattern requires the specific action of that muscle. The other patterns do not.
Adler SA, Beckers D, Buck M. PNF in Practice. New York, NY: Springer-Verlag; 1993: 111.

48. A patient is referred to physical therapy with a diagnosis of low back pain. Radiographic studies, including magnetic resonance imaging, have ruled out the presence of disc pathology. The patient reports continuous back pain that radiates upward toward the thorax and anteriorly into the abdominal region. The physical therapist should consider which of the following areas as a potential source of the discomfort? 

1. Dura mater 
2. Diaphragm 
3. Kidney 
4. Urinary bladder 

Correct Answer: 3 
Pain that is experienced in the thoracic spine can be caused by a variety of pathologic conditions. Low back pain can be either mechanical or non-mechanical in nature. Pathology in the kidney may refer pain to the lumbar spine (ipsilateral flank), or upper abdomen. Pathology in the urinary bladder refers to the suprapubic or thoracolumbar region. The diaphragm is innervated by C3, C4, and C5 with the pain normally confined to the C4 dermatome. The negative MRI has ruled out possible involvement of the dura mater.
Paz JC. In: Paz JC, Panik M, eds. Acute Care Handbook for Physical Therapists. Boston, MA: Butterworth-Heinemann; 1997: 504-505.

49. An adult patient who was involved in a motor vehicle accident has sustained multiple traumas, including fractured ribs on the right side. The patient is unconscious, intubated, and on a mechanical ventilator in the intensive care unit. Chest radiographs show the development of an infiltrate in the right lower lobe during the past 2 days. Rales and rhonchi are heard over the right lower lung fields. Which of the following chest physical therapy programs is MOST appropriate? 

1. Manual hyperventilation and suctioning while positioned on the left side 
2. Positioning supine for suctioning, followed by manual hyperventilation while positioned on the left side 
3. Suctioning, percussion, and vibration while positioned on the right side 
4. Positioning on the left side for deep breathing exercises only 

Correct Answer: 1 
In order to optimally clear the congestion that has developed in his right lower lobe, the patient would need to be positioned on the left side to allow gravity to help drain the secretions to the proximal airways. This patient is also unconscious and intubated so in addition to drainage, manual hyperinflation is necessary to provide increased ventilation which would help to mobilize secretions followed by suctioning to clear the secretions. Suctioning first then hyperventilating while in left-sidelying is less effective than the order suggested in Option 1. Positioning on the right side would not drain the right lower lobe and also since the patient is laying on the right side, the only place to percuss and vibrate would be the left side, which has no pathology. Finally positioning on the left side with deep breathing exercises will not be effective in mobilizing secretions, especially since the patient is unconscious and unable to actively perform deep breathing exercises.
Frownfelter D, Dean E. Principles and Practice of Cardiopulmonary Physical Therapy. 3rd ed. St. Louis, MO: Mosby;1996: 579-589.









50. A physical therapist reads an article on a muscle physiology study. The results of the study are shown in the graph. The therapist can BEST use the results of the study to explain the underlying rationale for which of the following interventions? 

1. The use of prolonged passive stretching to lengthen shortened connective tissue 
2. The use of plyometrics to enhance muscle power 
3. The use of closed chain versus open chain exercises to enhance co-contraction of muscles 
4. The use of hold-relax techniques for muscle stretching 

Correct Answer: 2 
Prolonged stretching does not depend on the length tension curve, but on the stress relaxation curve and golgi tendon organ. Plyometrics is a form of exercise designed to enhance muscle speed and power. The theory behind plyometrics relies on the stretch reflex (active contraction of a muscle or active tension) and the natural elastic components of skeletal muscle and its surrounding connective tissue (passive tension). The graph shows that as the muscle is stretched, the passive components are stretched and add to the total amount of tension that can be developed in a muscle. During plyometric exercises, a muscle is quickly stretched activating the muscle spindle and the stretch reflex, as well as stretching the connective tissue elements surrounding the muscle fibers. This combination produces a quicker, more forceful, contraction by the muscle. Closed chain exercise use is not explained by the length tension curve. The hold relax technique is thought to be mediated via the golgi tendon organ, not length tension.
Albert M. Eccentric Muscle Training in Sports and Orthopaedics. New York, NY: Churchill-Livingstone; 1991: 45-47.
Sullivan P, Markos PD. Clinical Decision Making in Therapeutic Exercise. Norwalk, CT: Appleton & Lange; 1995: 63-69.

51. In the early management of a patient with a partial peripheral nerve injury, the goal of the physical therapy intervention will MOST likely be to prevent: 

1. nerve degeneration. 
2. spasticity and increased muscle tone. 
3. muscle atrophy. 
4. contractures and adhesions. 

Correct Answer: 4 
The primary physical therapy goal in this case would be joint protection and the prevention of contractures and adhesions, usually through splinting. There is no evidence to suggest that physical therapy can prevent nerve degeneration and muscle atrophy, as they are the normal effects of a peripheral nerve lesion. Spasticity would not be present in a lower motor neuron injury.
Wadsworth CT. In: Myers RS, ed. Saunders Manual of Physical Therapy Practice. Philadelphia, PA: WB Saunders; 1995: 912.

52. A physical therapist is developing an educational program for individuals with lower extremity peripheral neuropathies due to diabetes. Which of the following information is MOST important for the physical therapist to recommend for the prevention of injury to the feet? 

1. Orthoses to support the extremity 
2. Use of proper footwear 
3. Moisturizing the skin to prevent dryness 
4. Exercise parameters 

Correct Answer: 2 
While all of the above would help prevent injury to a diabetic foot, the most important information to provide is about proper footwear. Ensuring properly fitting footwear would alleviate risk of skin breakdown as well as providing appropriate cushioning to the articular cartilage of the foot joints that are prone to injury from repetitive trauma and compression. Orthoses may not be necessary for all patients. Despite moisturizing the skin, if there is improper protection, skin breakdown can still occurs. Exercising within appropriate parameters will not be beneficial if the patient’s footwear is not supportive or properly fitting.
Goodman CC, Boissonnault WG. Pathology: Implications for the Physical Therapist. Philadelphia, PA: WB Saunders; 1998: 254-255.

53. Elevating a patient’s lower extremity for less than 1 minute produces a noticeable pallor of the foot, followed by delayed reactive hyperemia in a dependent position. These signs are indicative of: 

1. an intact circulatory system. 
2. arterial insufficiency. 
3. venous insufficiency. 
4. acute arterial occlusion. 

Correct Answer: 2
An intact circulatory system can sustain adequate blood flow to an elevated limb. A limb with arterial insufficiency may not be able to sustain adequate blood flow against gravity. The vasodilation, caused by local mechanisms, occurs in response to ischemia. An acute arterial occlusion creates pallor, but because blood flow is blocked, the reactive hyperemia is impaired. 
Wyngaarden JB, Smith LH. Cecil’s Textbook of Medicine. 17th ed. Philadelphia, PA: WB Saunders: 1985: 359-363.

54. While ascending stairs, an elderly patient leans forward with increased hip flexion. Which of the following muscles are being used to the best advantage with this forward posture? 

1. Rectus femoris 
2. Tensor fascia latae 
3. Gluteus maximus 
4. Lumbar paraspinal 

Correct Answer: 3 
The gluteus maximus is a hip extensor that is more active during resisted motions or when the hip is in flexion, especially in functional activities such as stair climbing. In this case, the patient flexes the hip placing the gluteus maximus on stretch increasing its ability to produce tension. Therefore, the patient is leaning forward to maximize the ability of the gluteus maximus to extend the hip during the stair climbing activity. The other muscles listed would not benefit as much from the increased hip flexion
Smith LK, Weiss EL, Lehmkuhl LD. Brunnstrom’s Clinical Kinesiology. 5th ed. Philadelphia, PA: FA Davis; 1996: 292.

55.. An inpatient physical therapy department has only 1 physical therapist and 1 physical therapist assistant on duty, due to staff illness. A patient with which of the following conditions and circumstances is MOST appropriate for the therapist to delegate to the assistant? 

1. Ataxia, who is undergoing a trial to determine an appropriate assistive device 
2. Hemiparesis, whose initial evaluation has not been completed 
3. Multiple sclerosis, who is receiving gait training with a rolling walker 
4. Alzheimer’s disease, who is easily agitated during the initial gait training 

Correct Answer: 3 
A physical therapist would be required to perform the initial evaluation, plan of care, re-evaluations, modifications to the plan of care, and discharge plans for the patient. The patient who is stable and on an existing program would be the most appropriate patient to be delegated to the assistant. Therefore, the patient who has multiple sclerosis and who is on an established gait training program would be the most appropriate. The other three patients are receiving their initial treatments.
Standards of Practice for Physical Therapy and the Criteria. In: American Physical Therapy Association. Guide to Physical Therapist Practice. Alexandria, VA: APTA; 1999: Appendix 2.

56. The brother of a patient who was recently discharged from the hospital’s outpatient physical therapy department telephones on the patient’s behalf to request a copy of the patient’s medical record. The physical therapist should explain to the patient’s brother that the medical record is the property of the: 

1. patient’s family and can be released to the brother upon written request. 
2. patient’s insurer now and that the request for a copy must be made in writing to the insurer. 
3. hospital and the patient and can be released only with written authorization from the patient. 
4. hospital and can be released only with written authorization from the patient’s physician. 

Correct Answer: 3 
The medical record is owned by the hospital subject to the patient’s interest in the information it contains. Unless restricted by state or federal law or regulation, a hospital shall furnish to a patient, or a patient’s representative parts of the hospital record upon request in writing by the patient or their representative. Option 3 is the only correct answer in this case, since it specifies that the patient must authorize the release of information.
Roach WH. Medical Records and the Law. 2nd ed. Gaithersburg, MD: Aspen; 1994: 96.

57. A physical therapist is examining a patient for possible lower extremity weakness. Passive range of motion is within normal limits. The patient is seated. When the patient tries to dorsiflex and invert the right foot, the patient is unable to move it through the full range of motion and is unable to take any resistance applied by the therapist. During the subsequent gait examination, the therapist should expect the patient to display which of the following? 

1. Normal gait with no deviations 
2. Increased right hip flexion during midswing phase 
3. Throwing the trunk backward on the right side shortly after heel strike (initial contact) 
4. Laterally bending towards the right side during midstance 

Correct Answer: 2 
The anterior tibialis shows poor (2/5) grade strength during the examination. Because this muscle is active during swing phase of gait and shortly after initial contact, one would expect to see gait deviations showing up at those times. Specifically, the anterior tibialis maintains dorsiflexion of the foot during swing phase and controls plantarflexion after initial contact. During swing phase, weakness of the anterior tibialis would cause the patient to increase the amount of hip and knee flexion to prevent toe drag. With weak hip abductors, the patient would be expected to have excessive pelvic rotation. To compensate the patient would laterally bend toward the weak (right) side during midstance to help prevent the excessive hip drop. Throwing the trunk backward after initial contact may be due to weakness of the hip extensors (gluteus maximus).
Norkin, CC, Levangie PK. Joint Structure and Function. Philadelphia, PA: FA Davis; 1992: 488-491.

58. A patient is referred to physical therapy with a diagnosis of chondromalacia patella. The physical therapist should decide to include quadriceps setting and straight-leg raises as part of the home exercise program, because: 

1. the vastus medialis muscle is primarily responsible for terminal knee extension.
2. quadriceps setting and straight-leg raises help improve patellar tracking. 
3. patello-femoral compression forces are increased when the knee is extended. 
4. the vastus medialis muscle is preferentially activated during a straight-leg raise.

Correct Answer: 2 
The patient’s symptoms would be aggravated by exercises that cause increases in patellofemoral joint reaction forces. One of the goals of treatment would be to promote proper patellar tracking. Isometric exercises such as quadriceps setting and straight leg raises (SLR) are most often used for strengthening with a diagnosis of chondromalacia patella because they produce smaller patellofemoral joint reaction forces than some other forms of quadriceps exercises, and thus are less painful. The vastus medialis is one of the muscles active during quadriceps setting. This muscle tends to pull the patella medially during contraction and thus acts as a dynamic medial stabilizer, which promotes proper patellar tracking. The vastus medialis is not preferentially recruited during a straight leg raise, nor is it the only muscle responsible for terminal knee extension. Patello-femoral compression forces are generally less during a SLR than when doing extension exercises with the knee bent.
Brotzman SB, Head P. In: Brotzman SB, ed. Clinical Orthopaedic Rehabilitation. St. Louis, MO: Mosby; 1996: 230-231.

59. Which of the following statements does NOT document patient outcome? 

1. The patient propelled his wheelchair independently after 4 weeks. 
2. The patient demonstrated independent performance of home program after 2 weeks. 
3. The patient walked 100 ft (30.5 m) with minimal assistance after 1 week. 
4. The patient attended physical therapy sessions 3 times/week for 2 weeks. 

Correct Answer: 4 
An outcome measure documents a desired performance or change in the patient’s condition over time. This may include a description of the patient’s function before, during and after intervention. Option 4 does not indicate a change or function, but is focused on attendance.
Nosse LJ, Friberg DG, Kovacek PR. Managerial and Supervisory Principles for Physical Therapists. Baltimore, MD: Williams & Wilkins; 1999: 226-227.

60. Which of the following is the MOST likely cause of a reduced vital capacity in a patient who has quadriplegia at the C5-C6 level? 

1. Decreased anterolateral chest expansion resulting from paralysis of the external intercostal muscles 
2. Inability of the patient to generate a negative intrapleural pressure secondary to a denervated diaphragm 
3. A relatively high resting position of the diaphragm resulting from paralysis of the abdominal muscles 
4. Reduced rib-cage elevation due to paralysis of the anterior scalene and sternocleidomastoid muscles 

Correct Answer: 1 
The rib cage would not be able to expand normally during inspiration due to weakness of the external intercostals muscles, which are innervated by thoracic nerve segments. With a spinal cord lesion at the C5-6 level, the diaphragm would still receive innervation from the phrenic nerve (C4). The anterior scalene (C4-6) would be partially innervated and the sternocleidomastoid (C2-3) would be fully innervated. The abdominal muscles would not be innervated since they receive their innervation from thoracic nerve segments. Paralysis of the abdominal muscles would cause the diaphragm to assume a low resting position.
Schmitz TJ. In: O’Sullivan SB, Schmitz TJ, eds. Physical Rehabilitation: Assessment and Treatment. 3rd ed. Philadelphia, PA: FA Davis; 1994: 539.

61. Instruction in energy conservation and joint protection should be provided to a patient with rheumatoid arthritis, because: 

1. the joints may be predisposed to damage by overuse. 
2. fatigue often masks joint pain. 
3. phagocytes remove more pannus in a resting joint. 
4. activity of the antigen-antibody complex is diminished with rest. 

Correct Answer: 1 
Rheumatoid arthritis is a chronic inflammatory disease that affects many body systems including the joint spaces. Destruction and subluxation of joints can occur over time secondary to the inflammation that occurs in the synovium. Fatigue is a common symptom that accompanies rheumatoid arthritis. Therefore, the patient must get enough rest to avoid excessive fatigue and to protect the joints from overuse damage. Therefore education on energy conservation and joint protection is essential to minimize joint deformity. However, the patient must realize that some activity is good for them to prevent contractures and to maintain strength and endurance. Fatigue and joint pain can and often do co-exist in patients with rheumatoid arthritis. The pannus in the joints is actually increased with rest. Rest does not appear to change the activity of the antigen-antibody complex.
Goodman CC, Boissonnault WG. Pathology: Implications for the Physical Therapist. Philadelphia, PA: WB Saunders; 1998: 663-667.

62. In which of the following is independence the MOST realistic long-term goal for a patient with a complete spinal cord lesion at C8? 

1. Effective cough technique 
2. Rolling from side to side 
3. Transfer from bed to wheelchair 
4. Walking with forearm crutches 

Correct Answer: 3 
This patient would have functional use of the triceps and finger flexors and would, therefore, be independent for all wheelchair transfers to all surfaces. Coughing would not be limited and would not be a goal. Rolling would be easily achievable as a short-term goal. Ambulation with forearm crutches would not be a reasonable expectation for this patient.
Nixon V. Spinal Cord Injury: A Guide to Functional Outcomes in Physical Therapy Management. Rockville, MD: Aspen; 1985: 51, 65, 120, 161.

63. To minimize skin irritation during functional electrical nerve stimulation, the physical therapist should use: 

1. lower intensity, larger interelectrode distance, and larger electrodes. 
2. lower intensity, larger interelectrode distance, and smaller electrodes. 
3. higher intensity, smaller interelectrode distance, and smaller electrodes. 
4. lower intensity, smaller interelectrode distance, and larger electrodes. 

Correct Answer: 1 
Several things can be done to decrease the current density and the possibility of skin irritation. These include decreasing the intensity of the stimulation, increasing the inter-electrode distance and the use of larger electrodes.
Gersh MR. In: Gersh MR, ed. Electrotherapy in Rehabilitation. Philadelphia, PA: FA Davis; 1992:183.

64. A patient is doing active and resistive exercises on a mat table in the physical therapy department. After 15 minutes, the patient becomes short of breath, begins coughing, and expectorates pink, frothy sputum. At this point, the physical therapist should first stop the treatment, then NEXT: 

1. assess vital signs, let the patient rest a few minutes with the feet elevated, and then resume with a less vigorous program. 
2. sit the patient up, assess vital signs, and call a nurse or physician for further instructions. 
3. lay the patient supine, transfer the patient to a stretcher, and return the patient to the nursing unit. 
4. lay the patient down flat, call for assistance, and begin cardiopulmonary resuscitation. 

Correct Answer: 2 
The presence of dyspnea and the pink, frothy sputum would suggest the presence of congestive heart failure and resultant pulmonary edema. Congestive heart failure can occur from poor cardiac muscle function as a result of myocardial infarction. Pulmonary edema occurs from the backflow of blood from the heart into the pulmonary vessels, increasing pulmonary capillary pressure. The increase in pulmonary capillary pressure increases fluid movement into the alveoli, which are normally dry. This leads to the presence of pink, frothy sputum that can be expectorated along with shortness of breath (dyspnea). Positions that increase blood flow to the heart, such as lying flat, will increase the signs and symptoms. Therefore, the patient should be positioned with the head up or as in this problem; the patient should be placed in a sitting position to help alleviate the symptoms. Laying the patient down flat, supine or with the legs elevated would exacerbate the patient’s problems.
Panik M, Paz JC. In: Paz JC, Panik M, eds. Acute Care Handbook for Physical Therapists. Boston, MA: Butterworth-Heinemann; 1997: 137-138.

65. A patient who sustained a mild cerebrovascular accident 3 weeks ago is being prepared by the physical therapist for discharge to home and an adult day program. To facilitate the discharge plan, the MOST appropriate health professional for the therapist to consult with is the: 

1. skilled nursing coordinator. 
2. occupational therapist. 
3. medical social worker. 
4. primary physician. 

Correct Answer: 3 
Medical social services deal with home situations, financial supports and acts as a resource director on behalf of the patient. Social services would be the most appropriate choice to help arrange and coordinate rehabilitative services for this patient while they are at home. Although nurses, occupational therapists, physical therapists and physicians may all be involved with direct patient care, they would not be the most appropriate for this level of discharge planning.
Lundrigan MM. In: May BJ, ed. Home Health and Rehabilitation: Concepts of Care. 2nd ed. Philadelphia, PA: FA Davis; 1999: 282-284.

66. A physical therapist examines a high school athlete in the training room. After removing the adhesive strapping from the athlete’s ankle, the therapist discovers that the athlete has developed an open weepy rash on the instep of the foot. The therapist should FIRST: 

1. apply moist heat to the foot. 
2. send the patient to a hospital emergency room. 
3. use more pre-wrap with the next joint taping. 
4. refer the patient to the team physician. 

Correct Answer: 4 
The open weeping rash may be infected. Skin infections are easily spread in a training room environment, and if not attended to correctly may infect more people. If an infection were suspected, the precise diagnosis would be made by means of a culture. Therefore, the most appropriate action would be to refer the patient to the team physician. Moist heat may increase the infection by increasing blood flow. An emergency room visit would not be indicated at this time. Occluding the wound with more pre-wrap creates an environment ideal for infection.
Scheinberg RS. In: Scuderi GR, McCann PD, Bruno PJ, ed. Sports Medicine. St. Louis, MO: Mosby; 1997: 105-111.

67. A patient who sustained a left transtibial amputation 2 years ago and a right transtibial amputation 3 weeks ago is being evaluated for possible walking with prosthesis. Which of the following factors is MOST relevant? 

1. Size of the right residual limb scar 
2. Length of the right residual limb 
3. Proficiency in previous prosthetic use 
4. Severity of phantom pain 

Correct Answer: 3 
The previously amputated left limb must function as the main support limb. Any treatment strategy for ambulation must ensure that the left limb is functioning optimally. While the other factors are relevant, the previous ability of the patient to ambulate is the greatest concern.
Mensch G, Ellis P. Physical Therapy Management of Lower Extremity Amputations. Rockville, MD: Aspen Publications; 1986: 107.

68. A prone standing board has been recommended for a 5 year-old child with severe hypotonia. The PRIMARY purpose for the use of a prone stander is to: 

1. promote weight bearing. 
2. reinforce an exaggerated positive support reflex. 
3. prevent hip dislocation. 
4. increase hip and knee range of motion. 

Correct Answer: 1 
Prone standers are used frequently for children who cannot achieve or maintain upright standing. The patient benefits from the physiologic changes associated with weight bearing and from the social and perceptual opportunity to see the environment from an upright position.
Kallenbacch AE. In: Tecklin J. Pediatric Physical Therapy. 3rd ed. Philadelphia, PA: JP Lippincott; 1999: 339-340.

69. A patient has difficulty palpating the carotid pulse during exercise. The patient should be instructed in alternate methods of self-monitoring, because repeated palpation is likely to result in: 

1. increasing the heart rate. 
2. decreasing the heart rate. 
3. an irregular heart rhythm. 
4. increasing systolic blood pressure. 

Correct Answer: 2 
Pressure receptors (baroreceptors) are present in the carotid sinus and these receptors respond to changes in blood pressure. An increase in blood pressure that is sensed by these receptors will stimulate the parasympathetic system to decrease the rate and force of contraction of the heart in order to help lower the pressure. Repeated palpation in the carotid sinus area may simulate an increase in blood pressure and cause this reaction. Therefore increased heart rate and blood pressure are incorrect. Irregular heart rhythms generally result from electrolyte imbalance and/or ischemia to the conduction system of the heart.
Brannon FJ, Foley MW, Starr JA, Saul LM. Cardiopulmonary Rehabilitation: Basic Theory and Application. 3rd ed. Philadelphia, PA: FA Davis; 1998: 31-32.

70. A patient spilled boiling water on the right arm when reaching for a pan on the stove. The forearm, elbow, and lower half of the upper arm appear blistered and red, with some subcutaneous swelling and pain on touch. To facilitate optimal function, the physical therapist should PRIMARILY emphasize: 

1. range of motion exercises to the right hand, wrist, and elbow. 
2. application of compression dressings. 
3. sterile whirlpool to the right upper extremity. 
4. splinting of the right upper extremity in full elbow extension. 

Correct Answer: 1 
The patient will be reluctant to move the limb because it is painful. ROM is essential to regain and maintain a functional ROM. In this case, the physical therapist’s primary goal is to restore function with ROM exercises. Compression dressings, whirlpool, and splinting, are all acceptable aspects of care that help in the restoration of ROM. However, functional ROM would be the primary goal.
Panik M, Jarrell-Gracious M. In: Paz JC, Panik M, eds. Acute Care Handbook for Physical Therapists. Boston, MA: Butterworth-Heinemann; 1997: 420.

71. A physical therapist plans to study the effect of cold compresses on passive range of motion in a group of 10 patients. The plan is to apply these compresses to the hamstring muscles 1 time/day for 5 days. Which of the following experimental designs is MOST appropriate for this type of study? 

1. For both the experimental and control groups, gather data from patient records.
2. For both the experimental and control groups, measure range of motion of both groups on day 5. 
3. For both the experimental and control groups, measure range of motion on days 1 and 5. 
4. For the experimental group, measure range of motion every day. For the control group, measure range of motion on days 1 
and 5. 

Correct Answer: 3 
The most appropriate design would be a two-group pretest-posttest. Both groups are measured at the same times, but only the experimental group receives treatment. Gathering information from patient records may not provide the same accuracy as if the researcher had performed all of the measurements directly. Measuring ROM only on day 5 would not account for any pretest differences between groups.
Currier DP. Elements of Research in Physical Therapy. 3rd ed. Baltimore, MD: Williams and Wilkins; 1990: 208-209.

72. During an examination of elbow strength using manual muscle testing, the patient supinates the forearm when attempting elbow flexion. Which of the following muscles is MOST likely doing the major part of the work? 

1. Biceps brachii 
2. Brachialis 
3. Supinator 
4. Brachioradialis 

Correct Answer: 1 
The biceps brachii is both an elbow flexor and supinator, and it is most effective as a supinator with the elbow flexed to about 90° (approximately the muscle testing position). The brachialis does not cause supination (only flexion). The supinator does not flex the elbow. The brachioradialis would move the forearm to a midposition rather than fully supinating it. Therefore, when the elbow both flexes and supinates the biceps brachii would be the most likely muscle causing this action.
Smith LK, Weiss EL, Lehmkuhl LD. Brunnstrom’s Clinical Kinesiology. 5th ed. Philadelphia, PA: FA Davis; 1996: 174-175.

73. A patient has pain, swelling, and tenderness over the medial border of the hand. The patient also shows changes in the color and temperature of the skin, hyperhidrosis, and progressive joint stiffness in the wrist and hand. The MOST likely cause of the patient’s signs and symptoms is: 

1. cervical disc disease. 
2. Raynaud’s phenomenon. 
3. Complex Regional Pain Syndrome. 
4. carpal tunnel syndrome. 

Correct Answer: 3 
All of these symptoms are indicative of reflex sympathetic dystrophy syndrome. Cervical disc disease does not produce swelling in the hand, color and temperature changes, or hyperhidrosis. Raynaud’s phenomenon results in pain, pallor, and coolness, but no hyperhidrosis. Although carpal tunnel syndrome can also show sympathetic nervous system abnormalities, compression of the median nerve would refer symptoms to the lateral border of the hand (thumb area). With carpal tunnel syndrome, the patient may also display thenar muscle weakness.
Saidoff DC, McDonough AL. Critical Pathways in Therapeutic Intervention. St. Louis, MO: Mosby; 1997: 52-65.

74. While a patient is walking in the parallel bars, the physical therapist observes that the pelvis drops down on the side opposite the stance extremity. This gait deviation is an indication of weakness of the hip: 

1. abductors of the swing extremity. 
2. adductors of the swing extremity. 
3. abductors of the stance extremity. 
4. adductors of the stance extremity. 

Correct Answer: 3 
The abductors are particularly active during the mid stance phase (single limb support) of gait to prevent the contralateral pelvis from excessive lateral tilting. Weakness of the hip abductors, particularly the gluteus medius, causes the hip to drop down on the side opposite the weakness. For example, weakness in the right gluteus medius would show up during stance phase on the right by excessive downward movement (lateral tilt) of the left pelvis. The hip adductors are active during terminal stance and through mid-swing and would not play a role in controlling the lateral rotation of the pelvis. The hip abductors show little activity during swing phase.
Perry J. Gait Analysis. Thorofare, NJ: Slack; 1992: 118-122, 258-259.

75. Which of the following structures is indicated by the arrow in the radiograph? 

1. Intertubercular groove 
2. Greater tubercle 
3. Lesser tubercle 
4. Coracoid process 

Correct Answer: 2 
The structure indicated by the arrow is the greater tubercle. The arm is internally rotated in this radiograph.
Magee DJ. Orthopedic Physical Assessment. 4th ed. Philadelphia, PA: Saunders; 2002: 297-301.

76. When treating a patient who has ankylosing spondylitis, the muscles requiring the MOST emphasis for strengthening exercises are the: 

1. pectorals. 
2. hip flexors. 
3. back extensors. 
4. abdominals. 

Correct Answer: 3 
Trunk ROM exercises and strengthening to minimize thoracic kyphosis are essential. Persons with ankylosing spondylitis tend to assume flexed postures. Disproportionately strong pectorals, hip flexors, or abdominals could worsen the kyphosis.
Goodman CC, Boissonnault WG. Pathology: Implications for the Physical Therapist. Philadelphia, PA: WB Saunders; 1998: 667-670.

77. A 3 month-old child has motor and sensory loss in the right upper extremity in the areas innervated by the C5 and C6 spinal nerves. The cause of this birth injury MOST likely is: 

1. hemiparesis. 
2. Erb-Duchenne paralysis. 
3. spinal cord injury. 
4. Klumpke’s paralysis. 

Correct Answer: 2 
A brachial plexus injury that affects the C5 and C6 nerve roots is termed Erb-Duchenne paralysis. A lesion affecting the C8 and T1 roots is termed Klumpke’s paralysis. Hemiplegia and spinal cord injury would be caused by trauma to the central nervous system and would not normally affect just the C5 and C6 nerve roots.
Magee DJ. Orthopedic Physical Assessment. 3rd ed. Philadelphia, PA: WB Saunders; 1997: 137.

78. A patient with multiple sclerosis is referred for physical therapy at home. Before going home, the patient requires training in bathing, dressing, and eating. Adaptive devices may be needed to accomplish this. The physical therapist should recommend that the patient be seen by: 

1. a social worker. 
2. an orthotist. 
3. an occupational therapist. 
4. a home health nurse. 

Correct Answer: 3 
The occupational therapist would be the most appropriate members of the health care team to teach the patient the needed self-care skills. The occupational therapist would also be able to provide the patient with information regarding modifications to the home environment that would increase the patient’s independence. The occupational therapist could teach the patient how to use adaptive devices and help with the fabrication of splints or self care aides.
May BJ. In: May BJ, ed. Home Health and Rehabilitation: Concepts of Care. 2nd ed. Philadelphia, PA: FA Davis; 1999: 11-12.

79. A patient with Parkinson’s disease has just been admitted to the rehabilitation unit. The patient is dependent in all transfers and requires moderate assistance of 1 person to walk 30 ft (9.1 m) with a standard walker. To facilitate good carry-over for activities, instruction of the family in transfers should occur: 

1. during a home visit after the patient is discharged. 
2. just prior to discharging the patient. 
3. early in the rehabilitation program. 
4. when the family feels ready to take the patient home. 

Correct Answer: 3 
The family should be involved in all stages of planning and treatment. Family involvement can shorten the rehabilitation process and the patient’s return to the community. It is important to have the family involved early in the rehabilitation process rather than wait until the patient is ready to be discharged.
Burton GU. In: Umphred DA, ed. Neurological Rehabilitation. 3rd ed. St Louis, MO: Mosby; 1995: 191.

80. A patient had final approval and checkout of a permanent prosthesis 2 weeks ago. The patient now reports to the physical therapist that the prosthesis is too heavy. The patient will benefit MOST from: 

1. an evaluation of the socket fit by the prosthetist. 
2. a prescription from the physician for a lighter-weight prosthesis. 
3. re-evaluation of hip strength by the physical therapist. 
4. adding a fork strap attachment to the prosthesis. 

Correct Answer: 3 
A complaint of heaviness likely reflects weakness. The complaint is not the fit or comfort of the prosthesis, so Option 1 is incorrect. The expense of obtaining a lighter prosthesis is not justified without determining if the solution is strengthening of muscle groups, which control the prosthesis. A fork strap attachment will assist with suspension, but will not reduce the weight of the prosthesis.
Mensch G, Ellis P. Physical Therapy Management of Lower Extremity Amputations. Rockville, MD: Aspen; 1986: 191-200.

81. A patient has a history of neck pain that is aggravated by long periods of sitting. The pain becomes progressively worse by evening. Range of motion and strength of the neck and shoulders are within normal limits. Sensation and reflexes are intact in both upper extremities. The patient has a forward head and excessive thoracic kyphosis. The MOST appropriate exercise program includes: 

1. stretching of the neck flexors and pectoral strengthening. 
2. upper trapezius strengthening and pectoral stretching. 
3. pectoral strengthening and rhomboid stretching. 
4. rhomboid strengthening and axial neck extension. 

Correct Answer: 4 
The history suggests that prolonged improper positioning of the cervical spine resulted in neck pain. A chronic forward head and kyphosis results in hyperextension in the upper cervical spine and excessive flexion in the upper thoracic spine. Further muscle length adaptation occurs with tight anterior muscles and stretched posterior muscles. Treatment should include correction of muscle weakness or imbalance. Rhomboid strengthening and axial neck extension are the only options that are both correct.
Greenfield B. In: Donatelli R, Wooden MJ, eds. Orthopaedic Physical Therapy. 2nd ed. New York, NY: Churchill Livingstone; 1994: 49.

82. Upon removal of a wet-to-dry dressing from a draining wound, a physical therapist observes that the skin immediately surrounding a patient’s wound is macerated. What should the therapist recommend for future wound care? 

1. Continue using the current dressing type. 
2. Make the dressing more absorbent. 
3. Leave the dressing on longer between dressing changes. 
4. Change to a pressure-type dressing. 

Correct Answer: 2 
Macerated tissue results from excessive moisture. A more absorbent dressing would soak up the excessive moisture and prevent the maceration.
Kloth LC, McColloch JM, Feedar JA. Wound Healing: Alternatives in Management. 2nd ed. Philadelphia, PA: FA Davis; 1995: 169.

83. A patient is performing a Phase I (inpatient) cardiac rehabilitation exercise session. The physical therapist should terminate low-level activity if which of the following values changes from resting level? 

1. The diastolic blood pressure increases to 120 mm Hg. 
2. The respiratory rate increases to 20 breaths per minute. 
3. The systolic blood pressure increases by 20 mm Hg. 
4. The heart rate increases by 20 bpm. 

Correct Answer: 1 
During Phase I (inpatient) cardiac rehabilitation, vital sign parameters with activity that warrant termination are: diastolic blood pressure of 110 mm Hg or greater, systolic blood pressure above 210 mm Hg or an increase greater than 20 mm Hg from resting and a heart rate that increases beyond 20 beats per minute above resting. The normal resting respiratory rate can range between 12 to 20 breaths per minute in adults, so an increase to 20 breaths per minute with low-level activity would not be a reason to terminate the activity.
Hillegass EA, Sadowsky HS. Essentials of Cardiopulmonary Physical Therapy. Philadelphia, PA: WB Saunders; 1994: 642-643.

84. A physical therapist is preparing to teach a patient who has Guillain-Barré syndrome to transfer from the wheelchair to a mat table. The patient weighs 150 lb (68 kg), has Poor (2/5) grade strength in both of the lower extremities, and Fair (4/5) grade strength in the left upper extremity. The right upper extremity has Normal (5/5) strength. Which of the following assisted transfers is MOST appropriate for the patient’s initial session? 

1. 2-person lift to the right side 
2. Sliding-board transfer to the right side 
3. A hydraulic lift from the wheelchair to the mat 
4. Full standing-pivot transfer to the right side 

Correct Answer: 2 
Because this patient has significant weakness in both lower extremities, a standing pivot transfer would not be safe. However, since the patient does have fair to normal upper extremity strength, and is not totally dependent, a hydraulic lift or two person lift would not be appropriate because it would not be enhancing the patient’s functional skills. The most appropriate transfer method would be for the patient to use a sliding board transfer to her strongest (right) side.
Minor MA, Minor SD. Patient Care Skills. 4th ed. Stamford, CT: Appleton & Lange; 1999: 249.

85. In physical therapy program planning for geriatric patients, an important age-related change that should be taken into consideration is: 

1. the inability to learn new motor tasks. 
2. decreased pain sensation. 
3. decreased motivation. 
4. the inability to select alternative movement strategies. 

Correct Answer: 2 
Neurological changes that are age related include decreased pain and vibratory sensation. The other changes listed do not normally occur with aging in the absence of pathology.
Lewis CB. Improving Mobility in Older Persons: A Manual for Geriatric Specialists. Rockville, Maryland: Aspen; 1989: 5.

86. To achieve maximum reduction of lymphedema following a mastectomy in the upper extremity by means of massage, it is MOST important that: 

1. local heat be applied before the massage. 
2. the upper arm be massaged before the forearm. 
3. the hand be massaged before the forearm. 
4. the massage strokes occur in a centrifugal direction. 

Correct Answer: 2 
Massaging the upper arm before the forearm helps reduce the proximal resistance of lymphatic flow, which will optimize the flow of fluid or edema from the distal areas. This has been referred to as the "uncorking effect". Local heat would cause vasodilation and may increase edema. Massaging the hand before the forearm would not allow the "uncorking effect" to occur. The massage strokes should be from distal to proximal (centripetal) direction.
Pfalzer LA. In: Myers RS, ed. Saunders Manual of Physical Therapy Practice. Philadelphia, PA: WB Saunders; 1995: 165-168.

87. A patient with a diagnosis of complete spinal cord lesion at C4-C5 demonstrates a weak, cough mechanism. Which of the following techniques would MOST effectively augment the patient’s coughing? 

1. Manual pushing against the upper abdomen 
2. Positioning the patient in prone 
3. Pursed-lipped breathing 
4. Interrupting the expiratory air stream 

Correct Answer: 1 
In order to cough effectively, the abdominal muscles must contract to expel air forcefully. This patient does not have this ability; therefore manual force against the abdomen would mimic the action of the abdominal muscles. The prone position would not assist the cough mechanism in this patient. Both pursed-lipped breathing and interrupting the expiratory air stream would decrease the force of exhalation and decrease cough effectiveness.
Irwin S, Tecklin JS. Cardiopulmonary Physical Therapy. 3rd ed. St. Louis, MO: Mosby; 1995: 596-597.

88. A patient is referred for recommendation regarding purchase of a wheelchair. The measurements of the patient while sitting are 16 in (40.6 cm) across the widest point of the hips and 18 in (45.7 cm) from the rear of the buttocks to the popliteal crease. Which of the following wheelchair dimensions are BEST suited for this patient’s needs? 

1. Seat width and seat depth of 18 in (45.7 cm) 
2. Seat width of 18 in (45.7 cm) and seat depth of 16 in (40.6 cm) 
3. Seat width and seat depth of 16 in (40.6 cm) 
4. Seat width of 16 in (40.6 cm) and seat depth of 18 in (45.7 cm) 

Correct Answer: 2 
The seat width should be slightly wider than the width of the widest body part, and the depth should come to within 1 inch of the popliteal fossa.
Atrice MB, Gonter M, Griffin DA, Morrison SA, McDowell SL. In: Umphred DA, ed. Neurological Rehabilitation. 3rd ed. St Louis, MO: Mosby; 1995: 527.

89. Which of the following objectives is MOST important prior to discharge, for a patient who has had a myocardial infarction? 

1. Ascend a flight of stairs before discharge. 
2. Perform prescribed exercises without angina. 
3. Return to normal daily activity level. 
4. Take a radial pulse reliably before discharge. 

Correct Answer: 4 
There is a normal, linear relationship between heart rate and exercise intensity in patients who are not on beta blocker medication or who are on non-rate responsive pacemakers. Therefore, teaching patients to take their pulse reliably before discharge is an important learning objective to ensure safety with exercise upon discharge. Ascending a flight of stairs before discharge is only appropriate if the patient needs to negotiate stairs at home, but not as important as pulse monitoring. Performing exercises without angina can be achieved by monitoring heart rate with exercise; therefore, taking a pulse is necessary. Returning to normal daily activity immediately after an MI is an unreasonable objective as it will take 6 to 8 weeks for the myocardium to heal; therefore, activity levels will need to be tempered in order to protect the heart while it is healing.
Collins SM. In: Paz JC, Panik M, eds. Acute Care Handbook for Physical Therapists. Boston, MA: Butterworth-Heinemann; 1997: 52-54.

90. A patient is undergoing a treadmill stress test. The appearance of abnormally wide, irregularly spaced QRS complexes on the electrocardiogram represents: 

1. ventricular depolarization. 
2. premature ventricular contractions. 
3. atrial fibrillation. 
4. atrial repolarization. 

Correct Answer: 2 
The QRS complex represents ventricular contraction. Atrial contraction is represented by the ‘p’ wave. When an area of the ventricle becomes irritable and develops an ectopic foci, the ventricle will depolarize prematurely before the normal conduction sequence (e.g., prior to SA node firing in the atria) and presents as a wide, irregularly spaced QRS complex. In normal ventricular depolarization, the QRS would be narrow and regularly spaced and atrial fibrillation would appear as the ‘p’ wave changes. Atrial repolarization occurs within the QRS complex and is not normally visible on EKG.
Hillegass EA, Sadowsky HS. Essentials of Cardiopulmonary Physical Therapy. Philadelphia, PA: WB Saunders; 1994: 384-386

91. Treatment of a patient with hemophilia who has a subacute hemarthrosis of the knee should INITIALLY include: 

1. active assistive range-of-motion exercise to the knee. 
2. instruction of the patient for weight bearing to tolerance. 
3. gentle resistive range-of-motion exercise to the knee. 
4. continuous immobilization of the knee in an extension splint. 

Correct Answer: 1 
This stage of hemarthrosis will mean that there is still some bleeding into the joint space, but not as extensively, therefore the patient will benefit from range of motion exercise to prevent contracture. The patient may need active-assist, as there may still be pain or edema in the joint that prevents independent performance of range of motion. The mechanical trauma of weight bearing to tolerance at this stage may impinge and damaged the pathologic synovium within the joint. Resistive range of motion is more appropriate when pain and swelling have subsided and no bleeding is occurring. Continuous immobilization in the extended position will promote contracture in the edematous knee.
Goodman CC, Boissonnault WG. Pathology: Implications for the Physical Therapist. Philadelphia, PA: WB Saunders; 1998: 389-394.

92. Which of the following home programs is MOST appropriate for a patient with chronic lateral epicondylitis? 

1. Using a forearm cuff to increase loading on the extensor tendons 
2. Performing exercises for wrist strength and stretching 
3. Administering iontophoresis with dexamethasone (Decadron) and lidocaine (Xylocaine) 
4. Doing friction massage of the brachioradialis tendon 

Correct Answer: 2 
Lateral epicondylitis is caused by overuse of the wrist extensors that originate on the lateral epicondyle of the humerus, especially the extensor carpi radialis brevis. If the lateral epicondylitis is at a chronic stage, conditioning of the extensor muscles and sustained grip activities will be most effective in long term management. A forearm cuff is thought to decrease the muscle loading. Iontophoresis would not be appropriate for a home program. Friction massage of the brachioradialis would not be appropriate since the extensor carpi radialis brevis muscle is usually the one affected.
Andrews JR, Wilk KE, Groh D. In: Brotzman SB, ed. Clinical Orthopaedic Rehabilitation. St. Louis, MO: Mosby; 1996: 75-79.

93. A physical therapist who works in a home health agency is treating a patient with diabetes mellitus. The patient tells the therapist that he is no longer taking his insulin. The physical therapist’s FIRST course of action should be to: 

1. instruct the patient in the proper technique for injection of insulin. 
2. contact the patient’s home health nurse. 
3. tell the patient’s family to report this information to the physician. 
4. have the patient perform a urine glucose test while the therapist is in the home. 

Correct Answer: 2 
Contacting the patient’s home health nurse is the appropriate first course of action because of the possible safety risk associated with the patient not taking their insulin. Injections and glucose testing would go beyond the scope of practice for physical therapy. Placing the responsibility on the family would not be appropriate because of the gravity of the situation.
Anemaet WK, Moffa-Trotter ME. The User Friendly Home Care Handbook. McLean, VA: Learn Publications; 1997: 11:11.

94. Which of the following methods is appropriate for handling a 1 year-old child with cerebral palsy who exhibits strong extensor tone in the trunk and extremities?

1. Carrying the child in a sitting position 
2. Carrying the child over one’s shoulder 
3. Keeping contact with the back of the child’s head 
4. Picking the child up under the upper extremities 

Correct Answer: 1 
The sitting position promotes visual attending, use of the upper extremities, and social interaction. A child who exhibits extensor posturing should be carried in a symmetric position that does not allow axial hyperextension and keep the hips and knees flexed.
Olney SJ, Wright MJ. In: Campbell S, ed. Physical Therapy for Children. Philadelphia, PA: WB Saunders; 1994: 498.

95. A physical therapist evaluates a 65 year-old patient with psoriasis for treatment with ultraviolet radiation. The patient’s history includes the following: total hip arthroplasty 1 year ago; penicillin allergy; calcium supplements since menopause; and daily tetracycline (Achromycin) use for the past 2 weeks for a urinary tract infection. Which of these factors should alert the therapist to use extra caution when applying ultraviolet light? 

1. Arthroplasty with a metal implant 
2. Daily tetracycline (Achromycin) use 
3. Penicillin allergy 
4. Long-standing calcium supplement use 

Correct Answer: 2 
Caution should be used in patients who are taking tetracycline, because the drug will enhance the effects of the ultraviolet radiation. Ultraviolet is not contraindicated in these patients, however the dosage of the ultraviolet would probably have to be adjusted to some lower level. The penicillin allergy, calcium supplements and metal implants would not be affected by the ultraviolet light. It should be noted that although ultraviolet radiation therapy is not common in today’s practice of physical therapy, therapists should nonetheless be aware of the precautions; and be able to advise patients who may be using ultraviolet radiation at home.
Weisberg J. In: Hecox B, Mehreteab TA, Weisberg J, eds. Physical Agents. Englewood Cliffs, NJ: Prentice Hall; 1994: 382.

96. Which of the following instructions is MOST appropriate for teaching a patient with C6 quadriplegia to transfer from a wheelchair to a mat? 

1. Keep your fingers extended to give a broader base of support. 
2. Rotate your head and shoulders in the same direction as the desired hip motion. 
3. Rotate your head and shoulders in the direction opposite to the desired hip motion. 
4. Keep both hands next to your knees to lock your elbows. 

Correct Answer: 3 
The position described creates the necessary force to move the lower body in this transfer given the level of the spinal cord injury. Finger extension against resistance would be difficult for a patient with C6 quadriplegia. The patient’s hands would be kept near the thigh or hips with one hand on the mat and one on the wheelchair.

97. A patient comes to physical therapy with pain in the lateral aspect of the proximal forearm. During the examination, the physical therapist tests the patient in the following position: 1) Supine with right upper extremity in the relaxed position 2) Left upper extremity in shoulder girdle depression, elbow extension, shoulder internal rotation, and wrist flexion. The patient’s symptoms are reproduced in this position and are relieved when the therapist removes the shoulder girdle depression component. The presence of which of the following conditions is an appropriate clinical interpretation of these findings? 

1. Cubital tunnel syndrome 
2. Pronator teres syndrome 
3. Radial tunnel syndrome 
4. Anterior interosseous syndrome 

Correct Answer: 3 
Cubital tunnel syndrome is ulnar nerve entrapment. The test as described is for the radial nerve. Tingling and paresthesias in the medial forearm and hand are typical findings with cubital tunnel syndrome. Pronator teres syndrome is compression of the median nerve. Although the patient may have pain on the radial side of the hand with pronator syndrome, it usually extends from the little finger (1st digit) to the index finger (4th digit) and does not result in lateral elbow pain. The test described is for the radial nerve. Radial tunnel syndrome involves compression of the deep branch of the radial nerve and presents with symptoms similar to lateral epicondylitis, i.e. poorly localized pain over the radial aspect of the proximal forearm. Symptoms are reproduced in the above test position. Anterior interosseous syndrome is compression of the anterior interosseous nerve, which is a branch of the median nerve. The median, not the radial, nerve tension test will provoke symptoms of anterior interosseous nerve syndrome.
Butler D. The Sensitive Nervous System. Adelaide, Australia: Noigroup Publications; 2000: 325-328. 

Dutton M. Orthopaedic Examination, Evaluation, and Intervention. New York, NY: McGraw-Hill; 2004: 544, 557, 558-561. 

Magee DJ. Orthopedic Physical Assessment. 4th ed. Philadelphia, PA: Saunders; 2002: 148.

98. A patient comes to physical therapy with generalized body pain. Which of the following criteria would BEST provide the physical therapist with discriminating information to distinguish between the presence of fibromyalgia and the presence of any other painful conditions in the patient? 

1. The existence of specific tender points 
2. Loss of sleep over the past 3 months 
3. A poor response to stretching exercises 
4. Visual disturbances and loss of balance 

Correct Answer: 1 
Tender points are the single most powerful way to distinguish the presence of fibromyalgia from other painful conditions in a patient. Non-restorative sleep, not loss of sleep, is a typical finding with fibromyalgia. Stretching exercises may be quite beneficial for patients with fibromyalgia. Visual disturbances and loss of balance do not typically occur with fibromyalgia, but may occur with myofascial pain syndrome.
Dutton M. Orthopaedic Examination, Evaluation, and Intervention. New York, NY: McGraw-Hill; 2004: 220-221.

99. A patient, who recently had an ankle injury, reports pain in the front of the ankle while walking. This pain is reproduced when, with the patient in sitting, the physical therapist grasps the patient’s heel and with the foot remaining in neutral introduces a lateral (external) rotation force to the lower extremity. The pain is not reproduced with inversion or eversion tests to the ankle. The patient MOST likely has which of the following injuries? 

1. Syndesmosis sprain 
2. Calcaneocuboid sprain 
3. 5th metatarsal fracture 
4. Tibial stress fracture 

Correct Answer: 1 
Option 1 tests the integrity of the tibiofibular syndesmosis. With a positive test result, the patient will have reproduction of symptoms and in this case, anterior ankle pain. External rotation of the tibia may potentially compress the calcaneocuboid joint, but would not stress the ligaments. A 5th metatarsal fracture would not present with anterior ankle pain. Option 4 does not load the tibia and/or muscles enough to provoke pain from a stress fracture.
Magee DJ. Orthopedic Physical Assessment. 4th ed. Philadelphia, PA: Saunders; 2002: 804.

100. The joint indicated by the arrow contains what soft tissue structure? 

1. Glenoid labrum 
2. Middle deltoid muscle 
3. Biceps brachii tendon 
4. Pectoralis minor muscle 

Correct Answer: 3 
The joint indicated (subacromial) in the radiograph contains several soft tissue structures including the biceps brachii tendon, supraspinatus tendon, and subacromial bursa. The other structures listed are not located in this area.
Dutton M. Orthopaedic Examination, Evaluation, and Intervention. New York, NY: McGraw-Hill; 2004: 411.

101. A long distance runner presents to physical therapy with insidious onset of lower leg pain. The patient’s examination results reveal weakness of toe flexion and ankle inversion. The physical therapist suspects vascular compromise associated with this presentation. Palpation at which of the following locations is MOST likely to reveal diminished arterial pulses in the patient? 

1. Dorsal aspect of the foot 
2. Posterior to the lateral malleolus 
3. Posterior to the medial malleolus 
4. Lateral aspect of the popliteal fossa 

Correct Answer: 3 
The dorsal aspect of the foot is the site for palpation of the dorsalis pedis pulse. The dorsalis pedis artery is not involved in posterior compartment syndrome. This palpation site of option 2 is most likely to access the peroneal artery. The peroneal artery is not involved in posterior compartment syndrome. The posterior tibial artery is the most likely to be involved in chronic posterior compartment syndrome, which is described in the question. This artery should be palpated posterior to the medial malleolus. Palpation in the popliteal fossa is a poor choice of palpation location in association with posterior compartment syndrome, which is described. The popliteal artery may be palpated here, but the compartment syndrome manifestations are distal to this site.
Palastanga N, Field D, Soames R. Anatomy and Human Movement: Structure and Function. 3rd ed. Boston, MA: Butterworth-Heinemann; 1998: 570.
Magee DJ. Orthopedic Physical Assessment. 4th ed. Philadelphia, PA: Saunders; 2002: 820.Reid DC. Sports Injury Assessment and Rehabilitation. New York, NY: Churchill Livingstone; 1992: 288-289.

102. A patient with low back and leg pain has been seen 2 times in outpatient physical therapy. Today, during the patient’s third visit, the therapist notes an audible slap of the foot at heel strike (initial contact). The patient also reports that there has been no leg pain since yesterday. Regarding testing, it is MOST important for the physical therapist to perform: 

1. Achilles’ tendon reflex testing. 
2. sensation testing of the sole of the foot. 
3. the sign of the buttock test. 
4. tibialis anterior strength testing. 

Correct Answer: 4 
Achilles’ (S1- S2) testing will not access the involved level (L4). Sole of the foot sensation testing (L5, S1) will not access the involved level (L4). As the sign of the buttock test is for space occupying lesions in the pelvis, hip pathology, or something that is non-musculoskeletal, it is not indicated at this time, based on the information provided about the patient. This patient demonstrates foot slap gait that is most likely associated with progression of nerve root compromise (i.e. increased disc protrusion). The therapist must assess the level/degree of compromise of the involved level, L4. Strength testing of the tibialis anterior will offer the most important information, as it is innervated by the root most likely compromised. The scenario indicates peripheralization of symptoms, which is considered worsening of the condition.
Dutton M. Orthopaedic Examination, Evaluation, and Intervention. New York, NY: McGraw-Hill; 2004: 29, 1171, 1182, 1307.

103. A patient comes to physical therapy with wrist pain following a fall on an outstretched hand. Between the tendons of the abductor pollicus longus and the extensor pollicus longus, the physical therapist identifies significant tenderness to palpation. The pain is not reproduced with contraction of the surrounding muscles. Which of the following diagnoses is MOST likely present in the patient? 

1. Extensor carpi radialis tendinitis 
2. Colles’ fracture 
3. Triangular fibrocartilage complex injury 
4. Scaphoid fracture 

Correct Answer: 4 
This injury was from a fall on an outstretched hand and is unlikely to be tendinitis. Colles’ fractures do not present specifically with tenderness in the snuffbox. A triangular fibrocartilage complex injury is central/medial (ulnar side) and does not present with snuffbox pain. The location of option 4 describes the anatomical snuffbox. Tenderness with palpation in the anatomical snuffbox suggests a scaphoid fracture, especially from a fall on an outstretched hand injury.
Dutton M. Orthopaedic Examination, Evaluation, and Intervention. New York, NY: McGraw-Hill; 2004: 599.

104. During the physical therapy examination of a patient with low back and leg pain, the physical therapist determines that the patient experiences centralization of symptoms with extension. Which of the following exercises is MOST appropriate for the physical therapist to introduce first? 

1. Supine partial sit-ups 
2. Prone opposite arm and leg lift 
3. Quadruped opposite arm and leg lift on a therapy ball 
4. Bilateral bridging with feet on a therapy ball 

Correct Answer: 2 
Partial sit-ups encourage flexion, which is not indicated. This patient has an extension bias and prone opposite upper extremity/lower extremity is the easiest/safest exercise to initiate when extension is a favorable direction. Quadruped opposite arm and leg lifts on a therapy ball and bilateral bridging with feet on a therapy ball encourage flexion, which is not indicated.
Dutton M. Orthopaedic Examination, Evaluation, and Intervention. New York, NY: McGraw-Hill; 2004: 1202-1203.

105. A patient, who has fallen 3 times while walking on uneven surfaces, comes to physical therapy for intervention. Examination reveals that the patient does not have difficulty maintaining balance with the eyes closed or with the introduction of visual conflict. Which of the following interventions is MOST appropriate for this patient? 

1. Instruct the patient to walk a straight line while glancing left and right. 
2. Instruct the patient in exercises to strengthen the gastroc-soleus muscle group.
3. Refer the patient to an orthotist for a custom ankle-foot orthoses fitting. 
4. Provide the patient with a cane for walking. 

Correct Answer: 4 
Instructing the patient to walk a straight line while glancing left and right would address a vestibular problem, specifically oscillopsia, but would not help a patient with a primarily somatosensory deficit, as described in the question. Instructing the patient in exercises to strengthen the gastroc-soleus muscle group would not provide protection against falling, although it may be an appropriate adjunct in a comprehensive therapy program. Referring the patient to an orthotist for a custom ankle-foot orthoses fitting may be included at some point in the intervention scheme, but the most important factor is immediate protection from falling, which is best addressed with a cane. This patient has a somatosensory deficit and while introducing somatosensory challenges is appropriate, the most important thing to do is provide them with a cane for balance and upper extremity proprioceptive input.
Schubert MC, Herdman, SJ. In: O’Sullivan SB, Schmitz TJ, eds. Physical Rehabilitation: Assessment and Treatment. 4th ed. Philadelphia, PA: FA Davis; 2001: 825, 836.
Wallace B. In: Bandy WD, Sanders B, eds. Therapeutic Exercise: Techniques for Intervention. Baltimore, MD: Lipincott Williams & Wilkins; 2001.

106. A physical therapist is analyzing data regarding functional loss in patients with fibromyalgia. The functional loss scale utilized has a normal distribution. The therapist finds that a score of 33 is 2 standard deviations below the mean, while a score of 55 is 2 standard deviations above the mean. This indicates that: 

1. 50% of subjects scored 33 and 55. 
2. 15% of subjects scored below 33. 
3. 95% of subjects scored between 33 and 55. 
4. 6.67% of subjects scored above 55. 

Correct Answer: 3 
In a normal distribution, the area between +/- 2 standard deviation units represents about 95% of the population. Greater than 50% of the scores fell between 33 and 55. Approximately 2.5% of the scores fell below 33 and approximately 2.5% of the scores fell above 55.
Currier DP. Elements of Research in Physical Therapy. 2nd ed. Baltimore, MD: Williams & Wilkins; 1984: 102.

107. A patient has had left Achilles’ tendinitis for 6 weeks. No structural abnormalities are evident upon examination of the patient. Which of the following supportive shoe inserts is MOST appropriate for this patient? 

1. Full-contact custom orthosis, left only 
2. Off-the-shelf heel lift, left only 
3. Off-the-shelf heel lifts, bilaterally 
4. Full-contact custom orthoses, bilaterally 

Correct Answer: 3 
Since this is a temporary intervention, the cost of custom orthoses is not warranted. Also, there should not be unilateral intervention, to eliminate any problems associated with inducing a leg-length difference temporarily. Temporary heel lifts may be added to decrease stress on the Achilles during treatment of tendinitis, but these should be used bilaterally, because there may be problems in inducing a leg-length difference, even temporarily. Since this is a temporary intervention, the cost of custom orthoses is not warranted.
Donatelli R, Brasel J, Brotzman, SB. In: Brotzman SB, ed. Clinical Orthopaedic Rehabilitation. St. Louis, MO: Mosby; 1996: 365.

108. A 70 year-old female patient reports the onset of midback pain after working in a garden for several hours. The patient reports constant pain, which increases with deep breathing, and demonstrates limited thoracic spine active range of motion in all planes. The patient has a 40-year history of smoking and long-term use of prednisone (Deltasone). Based on this history, which of the following pathologies is MOST likely the cause of the patient’s back pain? 

1. Thoracic compression fracture 
2. Lung cancer 
3. Coronary artery disease 
4. Abdominal aortic aneurysm 

Correct Answer: 1 
The data in the question are risk factors for osteoporosis and possible compression fracture. This patient could have weak bones due to long term steroid use, smoking, or being postmenopausal. Although lung cancer is likely with a history of smoking, lung cancer is unlikely because of the symptom onset after prolonged flexion and should not cause limited active range of motion. Coronary artery disease and an abdominal aortic aneurysm would not cause limited active range of motion or pain with deep breathing. The patient’s history does not increase her likelihood for having an abdominal aortic aneurysm.
Boissonnault WG, Goodman CC. In: Goodman CC, Fuller KS, Boissonault WG, eds. Pathology: Implications for the Physical Therapist. 2nd ed. WB Saunders; 2003: 876- 877.

109. A physical therapist is working with an outpatient who had a cerebrovascular accident and currently lives in an assisted-living facility. Which of the following statements is the MOST appropriate functional goal for this patient? 

1. The patient will be able to don an ankle-foot orthosis with assistance. 
2. The patient will independently walk 165 ft (50 m) with a straight cane from the bedroom to the cafeteria. 
3. The patient will have Normal (5/5) strength of the quadriceps muscles. 
4. The patient’s balance will improve to be able to independently stand on the involved lower extremity for 20 seconds. 

Correct Answer: 2 
Option 2’s goal is the only goal that is objective, measurable, and functional. The goals in options 1, 3, and 4 lack function or specific measurable objective information.
Kettenbach G. Writing SOAP Notes. Philadelphia, PA: FA Davis; 1995: 83-84, 88.

110. Which of the following positions is the correct end position in which to mobilize the radial nerve? 

1. Shoulder abduction, elbow extension, wrist extension 
2. Shoulder flexion, elbow flexion, wrist extension 
3. Shoulder abduction, elbow flexion, wrist flexion 
4. Shoulder extension, elbow extension, wrist flexion 

Correct Answer: 4 
The shoulder abduction, elbow extension, wrist extension position is for the median nerve. The shoulder flexion, elbow flexion, wrist extension position is for the ulnar nerve. Option 3 is incorrect, because the radial nerve stretch requires elbow extension, not flexion. Option 4 is the correct position for end-range gliding of the radial nerve.
Hall CM, Thein-Brody L. Therapeutic Exercise: Moving Toward Function. 2nd ed. Philadelphia, PA: Lippincott Williams & Wilkins; 2005: 719.

111. A physical therapist is working on transfers with a patient who had a brainstem cerebrovascular accident. The patient has ataxia in all 4 extremities and a high level of extensor tone in the lower extremities. The patient has fair to good trunk control. Which of the following transfers is BEST for this patient? 

1. Squat-pivot 
2. Sliding-board 
3. Standing-pivot 
4. Dependent tuck 

Correct Answer: 1 
A squat-pivot transfer avoids full lower extremity extension, thus minimizing lower extremity extensor tone. A sliding-board transfer is not appropriate with a high level of ataxia in the upper extremities. A standing-pivot transfer is not appropriate because it may increase the extensor tone in the lower extremities. A dependent tuck transfer does not encourage active patient participation or promote independence.
Hardie RJ, Rothwell JC. In: Greenwood R, Barnes M, Mcmillan T, Ward C, eds. Handbook of Neurological Rehabilitation. 2nd ed. New York, NY: Psychology Press; 2003: 159-161.

112. Which of the following modalities BEST addresses the cause of calcific tendinitis in the bicipital tendon? 

1. Sensory level interferential current at 80 Hz to 100 Hz 
2. Iontophoresis with acetic acid at 60 mA/minute 
3. High-volt pulsed electrical stimulation at 200 pps 
4. Diathermy with a parallel treatment set up 

Correct Answer: 2 
Sensory level interferential current at 80 to 100 Hz does not address the problem itself, but may address any related pain. Iontophoresis with acetic acid can address the cause of calcific tendinitis, not just the symptoms. High volt pulsed electrical stimulation at 200 pps does not address the problem itself, but may address any related pain. Diathermy with a parallel treatment set up is not the best choice, as deep heat will not address the pathology.
Behrens BJ, Michlovitz SL. Physical Agents: Theory and Practice for the Physical Therapist Assistant. Philadelphia, PA: FA Davis; 1996: 321.
Cameron MH. Physical Agents in Rehabilitation. 2nd ed. Philadelphia, PA: WB Saunders; 1999: 235-236.

113. A physical therapist is treating a patient with back pain with transcutaneous electrical nerve stimulation, using biphasic pulsed current and conventional transcutaneous electrical nerve stimulation parameters. The patient reports a painful response during the initial transcutaneous electrical nerve stimulation treatment. Which of the following treatment modifications is the MOST appropriate in this situation? 

1. Decreased electrode size 
2. Decreased pulse width 
3. Increased pulse rate 
4. Switch to a monophasic current 

Correct Answer: 2 
Decreasing the electrode size will increase current density and make the treatment more intense and uncomfortable. Decreasing the pulse width will make the treatment more comfortable, still affecting large fibers (A beta) without stimulating the A delta and C fibers (pain). Increasing the pulse rate while using pulsed current will not make the treatment any more comfortable. It has been proposed that a higher frequency is more comfortable with alternating current, but this is not true with pulsed current. Only switching to a monophasic current will not affect this patient’s comfort.
Mehreteab TA. In: Hecox B, Mehreteab TA, Weisberg J, eds. Physical Agents. Englewood Cliffs, NJ: Prentice Hall; 1994: 264, 266-268.

114. A patient with an L4 - L5 posterolateral herniated nucleus pulposus is MOST likely to have sensory deficits in which of the following locations? 

1. Medial knee 
2. Medial ankle 
3. Plantar aspect of the foot 
4. Dorsum of the great toe 

Correct Answer: 4 
The medial knee is the L3 dermatome. The medial ankle is L4 dermatome. The plantar foot is S1 or S2. A posterolateral disc bulge at L4- L5 will most likely affect the L5 nerve root and the dermatome for the L5 nerve root. The L5 dermatome includes the dorsal aspect of the great toe. The dorsal area of the great toe is consistently included in the L5 dermatome.
Dutton M. Orthopaedic Examination, Evaluation, and Intervention. New York, NY: McGraw-Hill; 2004: 186.

115. A physical therapist is working with a patient who has multiple medical issues and has just finished chemotherapy. Which of the following tests is MOST appropriate to measure changes in this patient’s endurance over time? 

1. 10-meter walk for time 
2. 6-minute walk 
3. Timed Up and Go 
4. Maximum VO2 assessment 

Correct Answer: 2 
The 10-m walk for time test addresses speed more than endurance. By definition, the 6-minute walk test is the only option that addresses endurance. The timed up and go test does not measure endurance. A Maximum VO2 assessment does not directly measure functional endurance.
Cahalin LP. In: DeTurk WE, Cahalin LP. Cardiovascular and Pulmonary Physical Therapy: An Evidence-based Approach. New York, NY: McGraw-Hill Medical Publishing Division; 2004: 258-259.

116. A patient had a brainstem stroke 2 months ago. The patient is currently able to independently walk 65 ft (20 m) over level surfaces with a straight cane and ascend stairs with minimum assistance. Which of the following activities would MOST appropriately challenge this patient’s balance during a physical therapy session? 

1. Ascending stairs using a single handrail 
2. Standing on 1 leg with eyes closed 
3. Walking over uneven terrain 
4. Walking 130 ft (40 m) with a straight cane 

Correct Answer: 3 
Option 1 is not a more difficult activity than the one the patient is currently able to perform. It is not challenging the patient. Option 2 addresses balance, but is not a functional activity. Option 3 is correct, as it challenges the patient, addresses endurance, and is safe for this patient. Option 4 progresses endurance, not balance.
Carr JH, Shepherd RB. Neurological Rehabilitation: Optimizing Motor Performance. Boston, MA: Butterworth-Heinemann; 1998: 174-176.

117. A 72 year-old patient reports bilateral calf pain experienced while walking. The pain has been gradually increasing, after an insidious onset 2 years ago. Sitting decreases the patient’s symptoms, and the patient’s past medical history is significant for smoking and diabetes. Which of the following pathologies is MOST likely the cause of the patient’s leg pain? 

1. Lumbar stenosis 
2. Gluteal artery claudication 
3. Herniated nucleus pulposus 
4. Deep vein thrombosis 

Correct Answer: 1 
Lumbar stenosis by definition causes lower extremity symptoms with extension activities and stenosis is more common in an older population. Gluteal artery claudication would cause buttock pain, not calf pain. This patient is relatively too old for a herniated nucleus pulposus. Most herniated nucleus pulposus do not cause bilateral symptoms. Nothing given in the patient’s history would increase the likelihood of a deep vein thrombosis.
Dutton M. Orthopaedic Examination, Evaluation, and Intervention. New York, NY: McGraw-Hill; 2004: 1213.

118. Which of the following pulmonary function tests measures the average total amount of air moved during inspiration of normal breathing? 

1. Vital capacity 
2. Inspiratory capacity 
3. Tidal volume 
4. Inspiratory reserve volume 

Correct Answer: 3 
Vital capacity is measured from maximum inhalation to maximum exhalation. Inspiratory capacity measures volume with maximal inhalation. Tidal volume measures total volume of air moved during either inhalation or exhalation over a specific period of time (usually 1 minute) and then divided by the ventilatory rate. Inspiratory reserve volume measures inspiratory volume beyond normal inspiration.
Sadowsky HS. In: Hillegass EA, Sadowsky HS, eds. Essentials of Cardiopulmonary Physical Therapy. Philadelphia, PA: WB Saunders; 1994: 416-418.

119. A patient who recently had bilateral leg amputations wants to have a ramp built to travel from the back deck of his house to the pool in his wheelchair. The vertical distance from the door to the ground level is 5 ft (1.5 m). Which of the following ramp specifications is BEST for this patient? 

1. 1 continuous ramp, 60 ft (18 m) long 
2. 1 continuous ramp, 30 ft (9 m) long 
3. 2 ramps, each 60 ft (18 m) long, connected by a level area 
4. 2 ramps, each 30 ft (9 m) long, connected by a level area 

Correct Answer: 4 
One continuous ramp that is 60 ft (18 m) long is too long of a run without a level area to rest. One continuous ramp that is 30 ft (9 m) long is too steep a slope. Two ramps, each 60 ft (18 m) long and connected by a level area is too long of a run without a level area to rest and unnecessarily long. The rise should not be greater than 30 in (76 cm) for any 1 run and should be 1 ft (.3 m) in length per each 1 in (2.5 cm) rise.
Minor MD, Minor SD. Patient Care Skills. 3rd ed. Norwalk, CT: Appleton & Lange; 1995: 476.

120. A new clinical evaluation tool is reported to have a specificity of 0.88, which means that this tool has a high percentage of: 

1. true positives. 
2. false positives. 
3. true negatives. 
4. false negatives. 

Correct Answer: 3 
A high percentage of true positives would be a measure of sensitivity. A high percentage of false positives would indicate low sensitivity. True negatives is correct as 0.88 is high and specificity is to correctly identify true negatives. A high percentage of false negatives would indicate low, not high, specificity.

121. The results of pulmonary function tests of a patient with which of the following diseases is LEAST likely to show increased residual volume? 

1. Atelectasis 
2. Bronchiectasis 
3. Chronic bronchitis 
4. Emphysema 


Correct Answer: 1 
Atelectasis occurs when 1 or more segments or a lobe of the lung is collapsed. It often occurs following thoracic or upper abdominal surgery, when the patient has a reduced total lung capacity, functional residual capacity, and residual volume. Bronchiectasis is an abnormal dilation of the bronchi and bronchioles and results in increased residual volumes. The primary symptom of chronic bronchitis is chronic productive cough and results in increased residual volume. Emphysema usually results in significantly increased residual volumes.
Hammon WE. In: Frownfelter DL, ed. Chest Physical Therapy and Pulmonary Rehabilitation: An Interdisciplinary Approach. 2nd ed. Chicago, IL: Year Book Medical Publishers; 1987: 90-114.

122. During the initial evaluation of a patient with right upper extremity pain, the physical therapist notes that the patient’s right scapula is significantly lower than the left scapula. Shortness in which of the following muscles on the right is MOST likely to lead to this patient’s postural presentation? 

1. Latissimus dorsi 
2. Upper trapezius 
3. Rhomboids 
4. Teres major 

Correct Answer: 1 
The latissimus dorsi depresses the shoulder girdle and can be short in patients with depressed scapulae. The upper trapezius, if short, would elevate the scapula. The rhomboids, if short, would adduct and downwardly rotate the scapula. The teres major, if short, would abduct the scapula.
Sahrmann SA. Diagnosis and Treatment of Movement Impairment Syndromes. St. Louis, MO: Mosby; 2002: 223.

123. A patient reports numbness and tingling on the lateral side of the right lower leg that sometimes extends into the top of the foot. Which of the following structures is MOST likely involved? 

1. Lateral sural cutaneous nerve 
2. L4 nerve root 
3. Superficial peroneal nerve 
4. S1 nerve root 

Correct Answer: 3 
The sensory distribution for the lateral sural cutaneous nerve includes the upper portion of the lateral lower leg, but does not extend into the foot. The L4 nerve root usually covers the anterior knee and down into the medial lower leg and foot. The information given in the question includes the sensory distribution of the superficial peroneal nerve. The S1 nerve root extends into the lateral portion of the foot, not the top portion of the foot.
Netter FH. Atlas of Human Anatomy. Summit, NJ: CIBA-GEIGY Corp.; 1989: 510-511.

124. A home health patient who recently had a 3-vessel coronary artery bypass graft describes experiencing bilateral lower extremity swelling, leg pain, and shortness of breath, especially when lying down. The patient MOST likely has which of the following diagnoses? 

1. Deep vein thrombosis 
2. Myocardial infarction 
3. Pulmonary embolism 
4. Congestive heart failure 

Correct Answer: 4 
A deep vein thrombosis corresponds to the leg pain and possibly swelling, but not shortness of breath. A myocardial infarction corresponds to shortness of breath, but typically does not result in swelling acutely. A myocardial infarction could result in the development of congestive heart failure, which would result in these symptoms, but this is a secondary result, not a primary result. A pulmonary embolism would result in shortness of breath, usually not changed by position, and typically cardiac arrest. Typical signs of congestive heart failure include dyspnea, paroxysmal nocturnal dyspnea, orthopnea, and peripheral edema.
Cahalin LP. In: Hillegass EA, Sadowsky HS, eds. Essentials of Cardiopulmonary Physical Therapy. Philadelphia, PA: WB Saunders; 1994: 128-131.

125. Which of the following outcomes is the highest expected functional outcome for a patient with a complete C7 spinal cord injury? 

1. Minimal assistance with transferring from the floor to the wheelchair 
2. Independent with wheelchair mobility on smooth surfaces 
3. Minimal assistance with transferring from supine to sitting 
4. Independent with ascending a curb in the wheelchair 

Correct Answer: 2 
These patients require maximal assistance for floor-to-wheelchair transfers as they don’t typically have the trunk control required to perform this task. Typically with C7 spinal cord injury, patients are able to perform wheelchair mobility on smooth surfaces independently. They may require an upright wheelchair to assist with trunk control. These patients usually are independent with supine-to-sit transfers; by rolling from side to side and getting their arms behind them, they are able to come to long sitting. They do not have the trunk control to perform a wheelie, which is required for ascending a curb in a wheelchair.
Atrice MB, Morrison SA, McDowell SL, Shandalov B. In: Umphred DA, ed. Neurological Rehabilitation. 4th ed. St. Louis, MO: Mosby; 2001: 493.

126. Which of the following symptoms is MOST typically associated with Guillain-Barré Syndrome? 

1. Periods of relapsing and remitting motor weakness 
2. Motor weakness in a peripheral nerve distribution 
3. Rapid development of progressive motor weakness 
4. Motor weakness in a spinal nerve distribution 

Correct Answer: 3 
Motor weakness in a peripheral nerve distribution is usually associated with multiple sclerosis, not Guillain-Barré Syndrome. Although Guillain-Barré Syndrome does affect the peripheral nerves, it tends to affect multiple nerves, not just a single peripheral nerve distribution. Rapid development of progressive motor weakness is very characteristic of Guillain-Barré Syndrome. Guillain-Barré Syndrome affects multiple peripheral nerves.
Hallum A. In: Umphred DA, ed. Neurological Rehabilitation. 4th ed. St. Louis, MO: Mosby; 2001: 387.

127. In splinting or immobilization, the functional position of the hand includes wrist: 

1. extension, phalangeal flexion, and abduction of the thumb (1st digit). 
2. extension, phalangeal extension, and abduction of the thumb (1st digit). 
3. extension, phalangeal flexion, and adduction of the thumb (1st digit). 
4. extension, phalangeal flexion, and adduction of the thumb (1st digit). 

Correct Answer: 1 
The functional position of the wrist and hand describes the position from which the optimal function is most likely to occur. This position is described as: (1) slight wrist extension, (2) slight ulnar deviation, (3) fingers flexed at the MCP, PIP and DIP joints and (4) thumb slightly abducted.
Hertling D, Kessler RM. Management of Common Musculoskeletal Disorders. 3rd ed. Philadelphia, PA: Lippincott; 1996: 260.

128. A patient with the diagnosis of astrocytoma malignancy is receiving physical therapy in her home. While sitting in a chair at her dining room table, the patient begins to have a grand mal seizure. The physical therapist lowers her to the floor. The MOST appropriate next step for the therapist is to: 

1. activate the emergency response system. 
2. protect the patient from biting her tongue by inserting a tongue blade. 
3. roll the patient on her side. 
4. extend the patient’s neck for airway clearance. 

Correct Answer: 3 
Dialing 911 is not the first step and may not be indicated until further assessment is completed after the seizure. There is no need to try and keep the patient from biting her tongue or to insert an object into the mouth, which could cause damage to the patient’s teeth. During a seizure, injury should be prevented. Rolling the patient on her side will help prevent aspiration. Option 4 is not an appropriate response to the patient having a seizure.
In: Beers MH, Berkow R, eds. The Merck Manual of Diagnosis and Therapy. 17th ed. Whitehouse Station, NJ: Merck & Co; 1999: 1405.

129. Which of the following assignments is MOST appropriate for a physical therapist to delegate to a volunteer? 

1. Restocking treatment booths with linens, ultrasound gel, and massage lotion 
2. Attending a patient who is on a tilt table while the therapist takes a phone call 
3. Transporting a patient who reports dizziness back to the patient's room 
4. Transferring a patient from the mat table to a wheelchair 

Correct Answer: 1 
The restocking of treatment booths with supplies is the only option which does not involve direct patient contact or care, and thus is the MOST appropriate activity to delegate to a volunteer. Although volunteers may at times be involved with patient care activities (i.e., transporting patients), the patients in the above situations pose a potential risk and would require supervision by someone other than a volunteer.
Walter J. Physical Therapy Management: An Integrated Science. St. Louis, MO: Mosby-Year Book; 1993: 149-150.

130. A patient who has rheumatoid arthritis comes to physical therapy with signs of muscle atrophy, ecchymosis, puffy cheeks, and a diagnosis of osteoporosis. This patient is MOST likely receiving a high dosage of: 

1. penicillin (Ampicillin). 
2. prednisone (Deltasone). 
3. acetylsalicylic acid (Aspirin). 
4. gold salts. 

Correct Answer: 2 
Prednisone is a glucocorticoid that exhibits these side effects. The primary side effect of penicillin is an allergic reaction such as skin rashes and difficulty breathing. The primary side effect of aspirin is gastritis. The primary side effect of gold therapy is diarrhea, irritation of oral mucosa and skin rashes.
Ciccone CD. Pharmacology in Rehabilitation. 2nd ed. Philadelphia, PA: FA Davis; 1996: 430-431.

131. A client on a weight-loss program has been walking 3 days/week for 15 minutes, for the past 3 weeks. To progress the exercise program, which of the following will MOST likely accomplish the weight-loss goal? 

1. Maintain the current walking speed and increase the duration to 30 minutes. 
2. Increase the walking speed and keep the duration at 15 minutes. 
3. Walk 4 days/week and decrease the duration to 10 minutes. 
4. Change from walking 3 days/week to jogging 1 day/week for 20 minutes. 

Correct Answer: 1 
The optimal exercise duration for achieving a weight loss with a walking program is 40 to 60 minutes of continuous aerobic activity. Therefore once a patient is safely tolerating 15 minutes, the best progression is to increase the duration while maintaining the same intensity or walking speed. Increasing walking speed should only be performed once the patient can consistently tolerate 20 to 30 minutes of exercise. Decreasing the duration while increasing the frequency of exercise would not accomplish the goal 40 to 60 minutes of continuous exercise. A patient who has been walking for only 15 minutes 3 times per week would not be ready to begin jogging and jogging 1 time per week would be too low of an exercise frequency in general to achieve any training benefit.
Hillegass EA, Sadowsky HS. Essentials of Cardiopulmonary Physical Therapy. Philadelphia, PA: WB Saunders; 1994: 723-724.

132. A patient comes to physical therapy 2 hours after a rugby game in which the patient sustained a forceful blow to the anterolateral lower leg. Which of the following physical therapy examination procedures is MOST important for the physical therapist to include? 

1. Ability of the patient to walk on the toes 
2. Integrity of the dorsalis pedis pulse 
3. Strength of the ankle everters 
4. Reactivity of the Achilles’ tendon reflex 

Correct Answer: 2 
Walking on the toes tests superficial posterior compartment, not superficial anterior compartment. The physical therapist must rule out an acute anterior compartment syndrome and the dorsalis pedis pulse is the only test given in the options that will give the therapist information about the function of the anterior compartment. Eversion strength would be involved in lateral compartment syndrome, not anterior compartment syndrome. Achilles’ reflex tests deep posterior compartment (tibial nerve), not anterior compartment.
Starkey S, Ryan JL. Evaluation of Orthopedic and Athletic Injuries. Philadelphia, PA: FA Davis; 1996: 123.

133. A physical therapist examines a patient lying supine with both lower extremities extended. The therapist notes that both of the patient’s medial malleoli are aligned with each other. The therapist asks the patient to assume the long-sitting position and notes the position of the malleoli as shown in the photograph. What is the MOST likely cause of this patient’s test results? 

1. Excessive femoral anteversion 
2. Anterior rotation of the innominate on the right 
3. Functional leg-length discrepancy due to left, lateral pelvic tilt 
4. Posterior rotation of the innominate on the right 

Correct Answer: 4 
Excessive femoral anteversion could cause toeing in, during standing. Anterior rotation of the innominate would cause shortening of the extremity. Pelvic tilt may cause a leg-length discrepancy with the longer leg being on the side of the tilt. When the malleoli are aligned in supine and then change with long sitting it is indicative of innominate rotation. In this case when the right malleolus shifted inferiorly (gets longer) it indicates a posterior rotation of the pelvis on the side of the longer leg.
Haynes WB, Rouben DP. In: Donatelli RA, Wooden MJ, eds. Orthopaedic Physical Therapy. 3rd ed. New York, NY: Churchill Livingstone; 2001: 411.

134. During the shoulder examination of a patient, a physical therapist notes the presence of a capsular pattern without radicular pain. To help establish the cause of the capsular pattern, the therapist should NEXT: 

1. perform axial compression on the cervical spine to check for nerve root compression. 
2. ask the patient if there has been any prior trauma to the shoulder joint. 
3. check for a painful arc during active range of motion. 
4. examine the shoulder for a rotator cuff tear. 

Correct Answer: 2 
A nerve root impingement commonly gives radicular pain. One of the most common causes of a capsular pattern in the shoulder is traumatic arthritis following injury to the shoulder. The capsular pattern may gradually develop over time. A painful arc is often associated with a non-capsular pattern such as seen with bursitis or tendonitis at the shoulder. A torn rotator cuff is associated with weakness and loss of active shoulder elevation.
Magee DJ. Orthopedic Physical Assessment. 4th ed. Philadelphia, PA: Saunders; 2002: 5-7. Ombregt L, Bisschop P, ter Veer HJ. A System of Orthopedic Medicine. Edinburgh: Churchill Livingstone; 2003: 305-307.

135. A physical therapist is considering the use of phonophoresis as part of an intervention plan. Which of the following steps is the correct FIRST step in the decision making process to use phonophoresis? 

1. Outline the therapeutic goals and outcomes. 
2. Select the appropriate coupling agent and medication. 
3. Decide on the dosimetry by choosing the appropriate mode and frequency. 
4. Determine if there are any contraindications. 

Correct Answer: 4 
Although all of options are involved in the decision making process, the determination of contraindications is the first thing to consider, since all other options are unnecessary if the patient has other pathologies that could be a contraindication.
Belanger A. Evidence Based Guide to Therapeutic Physical Agents. Baltimore, MD: Lippincott Williams & Wilkins; 2002: 250.

136. A physical therapist is assessing the lifting technique of a patient who has a history of back pain. The patient performs a lift by picking up a light weight from the floor, without bending the knees. During the lift, the therapist notes that the patient demonstrates excessive lumbar flexion. Limitation of which of the following measures is the STRONGEST contributor to this finding? 

1. Hamstring flexibility 
2. Gluteal muscle strength 
3. Abdominal muscle strength 
4. Hip flexor flexibility 

Correct Answer: 1 
Tightness of the hamstrings causes excessive lumbar flexion, because tight hamstrings restrict the amount of hip flexion. Weak gluteals allow more hip flexion. Weak abdominals show up in antigravity flexion of the trunk, not passive forward bending. Hip flexor flexibility is not a factor in forward bending.
Sahrmann SA. Diagnosis and Treatment of Movement Impairment Syndromes. St. Louis, MO: Mosby; 2002: 27-28.

137. A physical therapist is working with a patient who had an open repair of the rotator cuff 2 weeks ago. The patient requests a transfer to another facility to continue physical therapy. Which of the following measures should the patient expect to have achieved at this time? 

1. Active shoulder abduction to 30° 
2. Fair (3/5) supraspinatus strength 
3. Passive shoulder flexion to 60° 
4. Full passive range of motion 

Correct Answer: 3 
Active range of motion is not allowed until approximately 6 weeks. Supraspinatus strength testing is not allowed due to the active range of motion restriction. After open surgical repair of the rotator cuff, passive range of motion is allowed to tolerance generally around 100° of elevation. Full range of motion would not be achieved by this time; the cuff is still protected from stresses.
Brotzman SB, Wilk KE, eds. Clinical Orthopaedic Rehabilitation. 2nd ed. Philadelphia, PA: Mosby; 2003: 176-177.

138. A physical therapist is evaluating a patient who had a stroke and is exhibiting poor foot and ankle control. When asked to lift the foot more during the midswing phase of gait, the patient dorsiflexes the ankle with excessive eversion. Facilitory electromyographic biofeedback to which of the following muscles is MOST likely to help correct this problem? 

1. Extensor digitorum longus 
2. Tibialis anterior 
3. Fibularis (peroneus) brevis 
4. Flexor hallucis longus 

Correct Answer: 2 
The patient has too much eversion during the midswing phase. This is probably caused by too much activity in the extensor digitorum longus or extensor hallucis longus. To correct the situation, the tibialis anterior should be facilitated since it dorsiflexes and inverts but doesn’t cause eversion. The fibularis brevis, an everter, would cause plantarflexion of the foot rather than dorsiflexion. The flexor hallucis longus does not dorsiflex the ankle.
Perry J. Gait Analysis: Normal and Pathological Function. Thorofare, NJ: SLACK; 1992: 70-73.

139. A patient presents with weakness throughout the right lower extremity with normal strength on the left side. Sensory testing shows a loss of pressure sensation over the right thigh and leg, and a loss of pain and temperature sensation over the left thigh and leg. The patient has a positive Babinski’s sign present on the right. Which of the following associated findings is MOST likely to be found during further examination of this patient? 

1. The presence of clonus in the left ankle 
2. Marked atrophy in the right lower extremity muscles 
3. Spasticity in the left lower extremity 
4. Increased deep tendon reflexes on the right side 

Correct Answer: 4 
The presence of clonus would be on the right side, not the left side. Spasticity would be on the right side, not the left side. This is a case of hemisection of the spinal cord, Brown-Sequard syndrome. The lesion is on the right side of the spinal cord and it is an upper motor neuron lesion (Babinski’s sign) with damage to the corticospinal tract, posterior columns, and lateral spinothalamic tract. Upper motor neuron spinal cord lesions present with hyperactive tendon reflexes (in this case on the right side), clonus, and spasticity ipsilateral to the side of the lesion. Pressure sensation is lost on the ipsilateral side and pain and temperature are lost contralateral to the side of the lesion due to the crossing of those fibers in the spinal cord.
Blumenfeld H. Neuroanatomy Through Clinical Cases. Sunderland, MA: Sinauer; 2002: 66, 231, 235.

140. A patient presents with sudden onset of weakness of the facial muscles on the right side. The patient is unable to wrinkle the forehead, smile, pucker the lips, or wrinkle the nose. There is an absence of tearing in the patient’s right eye, diminished taste sensation on the right side of the tongue, and dryness of the mouth. The patient’s corneal reflex is absent on the right, but normal on the left, and pinprick and temperature sensation are normal on both sides of the face. This presentation is characteristic of which of the following conditions? 

1. Trigeminal neuralgia 
2. Bell’s Palsy 
3. Left cortical cerebrovascular accident 
4. Oculomotor nerve damage 

Correct Answer: 2 
Trigeminal neuralgia causes facial pain. All of the signs and symptoms in the question are indicative of a partial nerve lesion affecting the facial nerve. This type of lesion is Bell’s Palsy. A left cerebrovascular accident affects only the lower facial muscles below the eye. Damage to the oculomotor nerve causes paralysis of 1 or more of the muscles that moves the eyeball.
Blumenfeld H. Neuroanatomy Through Clinical Cases. Sunderland, MA: Sinauer; 2002: 482-484.

141. A patient was injured in a hunting accident, sustaining a gunshot wound to the spine in the area of L1. The patient has weakness of the left lower extremity and inability to move the knee, ankle, or foot. The patient’s patellar tendon and Achilles’ tendon reflexes are increased on the left side. There is also loss of proprioception in the patient’s left ankle and knee, a positive Babinski’s sign on the left side, and diminished sensation to pinprick and temperature changes in the right thigh, leg, and foot. Results of all of the patient’s cranial nerves’ tests are normal. These findings are consistent with which of the following injuries? 

1. Complete severance of the spinal cord 
2. Injury to the left anterior horn of the spinal cord 
3. Injury to the left side of the spinal cord 
4. Injury to the central area of the spinal cord 

Correct Answer: 3 
Complete severance of the spinal cord causes motor and sensory loss on both sides of the body. A lesion of the anterior horn cells causes a lower motor neuron problem and hyporeflexia. The signs and symptoms presented in the question point to a hemisection of the spinal cord on the left side, which gives rise to ipsilateral motor and proprioception loss and contralateral loss of pain and temperature. Also present are hyperreflexia and a positive Babinski’s sign on the left side. A lesion of the central spinal cord usually spares the motor tracts.
Blumenfeld H. Neuroanatomy Through Clinical Cases. Sunderland, MA: Sinauer; 2002: 280.

142. Following anterior cruciate ligament reconstruction, the advantage of closed kinetic chain exercises over open kinetic chain exercises is PRIMARILY the: 

1. decrease in recruitment of the quadriceps muscles. 
2. decrease in hamstring force during knee extension. 
3. relative ease of isolation of pure quadriceps muscle activity. 
4. relative reduction of an anterior drawer effect at the knee. 

Correct Answer: 4 
Closed chain exercises may produce more force in the quadriceps. They also produce more hamstring force than open chain exercises. They cause co-contraction of agonist and antagonist muscles, disallowing isolated contraction of the quadriceps. Closed kinetic chain exercises for the knee are generally thought to produce a co-contraction of the quadriceps and hamstrings, an effect which is not associated with open chain knee extension exercises. The co-contraction causes the hamstrings to restrict anterior displacement of the tibia on the femur (anterior drawer effect) and puts less strain on the anterior cruciate ligament.
Brotzman SB, Wilk KE, eds. Clinical Orthopaedic Rehabilitation. 2nd ed. Philadelphia, PA: Mosby; 2003: 269-270.

143. A physical therapist is teaching a patient who has recently undergone knee surgery to descend stairs without a railing and using a cane. The patient is partial weight-bearing on the right. Which of the following instructional methods should the physical therapist use to teach the patient this activity? 

1. The therapist stands behind the patient, and the patient descends with the cane and left leg first. 
2. The therapist stands beside the patient, and the patient descends with the right leg first and then the cane. 
3. The therapist stands below the patient, and the patient descends with the cane and right leg first. 
4. The therapist stands behind the patient, and the patient descends with the left leg first. 

Correct Answer: 3 
Neither options 1, 2, nor 4 is safe or appropriate. When descending stairs, it is generally best for the therapist to be positioned below the patient as protection from falling. However, the patient should always descend stairs leading with the involved leg, the right leg in this case, and the cane.
Pierson FM. Principles and Techniques of Patient Care. 2nd ed. Philadelphia, PA: WB Saunders; 1999: 236-239.

144. A physical therapist is reviewing the chart of an inpatient, who is right-handed, before doing an evaluation. The therapist finds that the patient’s angiogram shows a complete blockage of the left middle cerebral artery. Which of the following is the therapist MOST likely to find upon performing the evaluation? 

1. Hemiparesis in the left upper extremity 
2. Presence of hemineglect syndrome 
3. Left homonymous hemianopsia 
4. Presence of expressive aphasia 

Correct Answer: 4 
Left hemiparesis occurs with a blockage of the right middle cerebral artery. Hemineglect syndrome is much more common in right cerebral hemisphere lesions. Left homonymous hemianopsia occurs with a blockage of the right middle cerebral artery. Complete blockage of the left middle cerebral artery would most likely cause a problem with expressive speech (Broca’s aphasia) especially in a right-handed individual.
Blumenfeld H. Neuroanatomy Through Clinical Cases. Sunderland, MA: Sinauer; 2002: 376-377.

145. A physical therapist is interviewing a patient who awoke this morning with a slight loss of taste sensation, inability to close the right eye along with dryness in that same eye. The patient has had a slight headache the last few days and some mild pain around the right ear. Which of the following questions that is consistent with these signs and symptoms should the therapist ask the patient? 

1. “Are you able to smile?” 
2. “Do you have pain over your right jaw?” 
3. “Are you able to chew food?” 
4. “Do you have difficulty swallowing?” 

Correct Answer: 1 
All of the questions are important in history taking; however the suspected diagnosis is Bell’s Palsy caused by a lesion of the facial nerve. The patient would most likely have weakness in the muscles of facial expression including smiling. Asking the patient to smile would most likely show an asymmetrical smile with weakness on the right side. Jaw pain and inability to chew food would be more likely due to a lesion of the trigeminal nerve. Inability to swallow may be caused by damage to cranial nerves IX, X or XII.
McKenzie RA, May S. The Lumbar Spine: Mechanical Diagnosis and Therapy Volume 2. Minneapolis, MN: Spinal Publications; 2003: 385-386.

146. A physical therapist is gait training a patient who suddenly becomes weak and falls to the floor. The patient is now unresponsive. The therapist should FIRST: 

1. open the patient’s airway. 
2. activate the emergency response system. 
3. quickly start breathing assistance. 
4. begin chest compressions. 

Correct Answer: 2 
Although all of the options are steps in the cardiopulmonary resuscitation guidelines, based on the presented signs and symptoms, activation of the emergency response system is the first thing the therapist should do, as the patient is located within a hospital setting.
American Heart Association. Guidelines 2000 for Cardiopulmonary Resuscitation and Emergency Cardiovascular Care. Dallas, TX: American Heart Association; 2000: 102: 22-59.

147. During the gait evaluation of a patient who has a transfemoral prosthesis, the physical therapist notices that the patient laterally bends excessively towards the prosthetic side during midstance phase. Which of the following is MOST likely to cause this gait deviation? 

1. A prosthesis that is too short 
2. A prosthetic socket that is too small 
3. Inadequate prosthesis suspension 
4. A locked knee unit 

Correct Answer: 1 
A prosthesis that is too short causes a patient to laterally bend towards the prosthetic side during stance phase. Each of the conditions given in options 2, 3, and 4 would make the prosthesis seem too long and none would cause the problem described.
Edelstein JE. In: O’Sullivan SB, Schmitz TJ, eds. Physical Rehabilitation: Assessment and Treatment. 4th ed. Philadelphia, PA: FA Davis; 2001: 666.

148. During the evaluation of a patient’s balance, the physical therapist gently pushes the patient backwards slightly and observes how the patient recovers from the perturbation. What strategy is the patient MOST likely to use to correct for this slight perturbation? 

1. Knee 
2. Hip 
3. Ankle 
4. Stepping 

Correct Answer: 3 
Regarding options 1, and 2, for larger perturbations, individuals utilize hip and knee muscles to recover the balance. Regarding option 3, for slight perturbations, most individuals use an ankle strategy. Ankle musculature is used to control the perturbation and recover the balance. Regarding option 4, if the perturbation is strong enough to cause the individual’s center of mass to move outside the base of support, a stepping strategy would be employed by taking a step and increasing the size of the base of support.
Shumway-Cook A, Woollacott MH. Motor Control: Theory and Practical Applications. Baltimore, MD: Williams & Wilkins; 1995: 126-130.

149. A patient is being seen by a physical therapist following a mild myocardial infarction. The goal is to increase the patient’s exercise endurance for return to work. The patient is otherwise healthy and is taking propanolol (Inderal) 1 time/day. The therapist is setting the patient up on a daily home exercise program utilizing a stationary bike and treadmill walking. The therapist should teach the patient to use which of the following methods to monitor exercise intensity during exercise sessions at home? 

1. Heart rate 
2. Blood pressure 
3. Perceived exertion 
4. Respiration rate 

Correct Answer: 3 
Normally, heart rate monitoring is the best method for monitoring exercise intensity. However, since the heart rate is controlled, it should not be used. Blood pressure is a measure of intensity, but is not practical in this case. The effect of the propanolol (Inderal), a beta blocking agent, is to control heart rate and decrease the workload on the heart. Perceived exertion (Borg’s scale) is the preferred method to monitor exercise intensity for a patient taking beta blocking agents. Respiration rate may not be an accurate indicator of exercise intensity and cardiac workload.
Paz JC, West MP. Acute Care Handbook for Physical Therapists. 2nd ed. Boston, MA: Butterworth-Heinemann; 2002: 66-67.

150. A patient who is a waitress reports weakness in the right hand that causes her to drop dishes at work whenever she is distracted. The patient also reports wrist and hand pain and numbness over the thenar eminence, both of which occur during the day and at night. The patient’s thenar muscle strength is Poor (2/5) throughout. Which of the following long-term physical therapy goals is MOST appropriate for this patient? 

1. The goal is to increase tactile sensitivity in 4 weeks, so that the patient is able to hold objects and not drop dishes at work. 
2. The patient will demonstrate decreased pain in the wrist in order to be able to sleep at night. 
3. The long-term goal is to increase strength in the thenar muscles and decrease pain in the wrist back to normal levels. 
4. The patient will demonstrate thenar muscle strength of Good (4/5) grade in 6 weeks and be able to hold objects in hand while at work. 

Correct Answer: 4 
A well written goal includes an audience (who), behavior (what will be done), condition (under what circumstances), and degree (amount of change). The only option that includes all of these conditions is, “The patient (who) will demonstrate thenar muscle strength (behavior) of Good (4/5) grade (degree) and be able to hold objects in hand while at work (condition). Options 1, 2, and 3, are each missing 1 or more of the required elements.
Kettenbach G. Writing SOAP Notes. Philadelphia, PA: FA Davis; 1995: 83-85.

151. A physical therapist places a patient on a resistance training program to increase strength in both lower extremities. The mode of exercise is a double-leg press unit using free weights. After 1 week, the patient shows a 10-lb (4.5-kg) increase in the amount of weight able to be lifted. What is the MOST likely cause of the patient’s increase in strength? 

1. Muscle fiber hypertrophy 
2. Neurological adaptation 
3. Hyperplasia of the muscle fibers 
4. Increase in the amount of actin and myosin 

Correct Answer: 2 
Strength increase in muscle is due to a number of factors including neurological adaptation and muscle fiber hypertrophy with an increase in actin and myosin. Hyperplasia in humans is still controversial. Long-term changes in muscle strength are due to all of the above factors. However, short-term changes, such as 1 week, are most likely to due to neurological factors such as more efficient motor unit recruitment, autogenic inhibition, and more efficient co-activation of muscle groups.
Wilmore JH, Costill DL. Physiology of Sport and Exercise. 3rd ed. Champaign, IL: Human Kinetics; 2004: 90-97.

152. A physical therapist is working with a patient who has a complete T6 spinal cord injury. The therapist has the patient perform seated push-ups on a mat by having the patient push down on the mat with both upper extremities and attempting to lift the buttocks off the mat. Which muscle group is being trained during this activity? 

1. Quadratus lumborum 
2. Internal obliques 
3. Latissimus dorsi 
4. External obliques 

Correct Answer: 3 
The Quadratus lumborum, internal obliques, and external obliques are all innervated below the level of the lesion. Sitting push-ups are often used as preparation for gait training and transfers. The push-ups are done to strengthen the muscles that “hike the pelvis” or lift the buttocks from the mat in a seated position. In a patient with a complete spinal cord injury at the T6 level the only muscle group that is still capable of lifting the pelvis is the latissimus dorsi, which is innervated by the cervical roots C6 - C8. The therapist is attempting to strengthen these muscles because they are capable of lifting the pelvis in the absence of the erector spinae and abdominal musculature which are innervated below the level of the lesion.
Schmitz TJ. In: O’Sullivan SB, Schmitz TJ, eds. Physical Rehabilitation: Assessment and Treatment. 4th ed. Philadelphia, PA: FA Davis; 2001: 903.

153. A physical therapist is treating a patient who has been diagnosed with lumbar spinal stenosis. The patient has central back pain for which the therapist is using mechanical lumbar traction to reduce. Which of the following traction positions is MOST appropriate for the therapist to use for this patient’s condition? 

1. Right sidelying with the knees flexed 
2. Supine with the knees flexed 
3. Left sidelying with the knees straight 
4. Prone with no pillows under the hips 

Correct Answer: 2 
Since the symptoms are centrally located, there is no need to use sidelying. Spinal stenosis involves a narrowing of the spinal canal. The canal is opened somewhat during sitting or positions of lumbar flexion. The canal is narrowed during lumbar extension. The best position for this patient is supine with the knees flexed. Left side lying with the knees straight and prone with no pillows under the hips promote extension. The canal is narrowed during lumbar extension.
Saunders HD. Evaluation, Treatment, and Prevention of Musculoskeletal Disorders. 3rd ed. Bloomington, MN: Saunders Group Company; 1993: 283.

154. A physical therapist is treating a patient who had knee surgery following a car accident. The patient is suing the driver of the other car and has hired an attorney. The patient is coming into physical therapy as an outpatient 3 times/week. After the third session, the patient’s niece calls and wants to know how much weight her aunt can bear on the leg. The therapist should: 

1. have the aunt’s attorney call the niece. 
2. describe the weight-bearing limits in general terms. 
3. have the niece call the medical records office. 
4. ask the niece to visit directly with her aunt. 

Correct Answer: 4 
A patient must give informed consent regarding information about the treatment. The best alternative for the therapist is to have the niece visit directly with the aunt or have the niece attend a treatment session with the aunt’s permission. Informed consent can only be given by the patient, unless the patient is incapable of doing so. In this case, there is nothing to suggest that the patient is unable to give her own consent.
Scott RW. Foundations of Physical Therapy: A 21st Century-focused View of the Profession. New York, NY: McGraw-Hill Medical Publications Division; 2002: 187-188.

155. When examining a patient with right facial weakness, the physical therapist notes the presence of a right Bell’s phenomenon. Which of the following additional findings is MOST likely to also be present? 

1. Decreased taste sensation on the tongue 
2. Abnormal tone of the right limbs 
3. Ptosis of the unilateral eyelid 
4. Loss of pain and temperature sensation across the lower jaw 

Correct Answer: 1 
The facial nerve, while primarily motor, does carry some sensations, particularly taste sensation from the anterior two-thirds of the tongue. Abnormal tone is associated with a supranuclear (central) lesion. Supranuclear lesions produce contralateral voluntary lower facial paralysis, so weakness would be on the other side. A Bell’s phenomenon is not present with central lesions. Ptosis or sagging of the eyelid occurs with weakness of the levator palpebrae muscle. The levator palpebrae muscle is innervated by the oculomotor nerve (III). Pain and temperature sensation from the face (jaw) is carried by the trigeminal nerve (V).
Wilson-Pauwels L, Akesson EJ, Stewart PA, Spacey SD. Cranial Nerves in Health and Disease. 2nd ed. London: BC Decker; 2002: 128-135.

156. A physical therapist is examining a patient who sustained a right tibial fracture 1 year ago and is now fully healed. The physical therapist’s measure of the length of the patient’s right tibia is 14 in (36 cm) and left tibia, 15 in (38 cm). Tightness is MOST likely to be found in which of the following muscles in this patient? 

1. Left tensor fascia lata 
2. Left piriformis 
3. Right gluteus medius 
4. Right adductor magnus 

Correct Answer: 3 
The left tensor fascia lata would be chronically stretched, rather than shortened. The piriformis is an extensor and external rotator of the hip, so not as likely to be affected. The right leg is shorter, so in standing, the patient’s right hip should be lower than the left. This relatively places the right lower extremity in abduction and the left lower extremity in adduction. This chronic posturing would mostly likely cause tightness in the right hip abductors and the left hip adductors. Of all the options, the right gluteus medius muscle is the most likely to be tight, because it is a right hip abductor. The right adductors would be in a chronically lengthened position, so not as likely to be affected.
Hall CM, Brody LT. Therapeutic Exercise: Moving Toward Function. Philadelphia, PA: Lippincott Williams & Wilkins; 1999: 390-391.

157. A patient with an impairment that limits hip flexion to 105° will benefit MOST from which of the following assistive devices and modifications to accomplish activities of daily living? 

1. Elevated chair-seat height 
2. Long-handled reacher 
3. Decreased rise in stair height 
4. Sock aid 

Correct Answer: 4 
Normal sitting requires about 105° of hip flexion, so an elevated chair would not be necessary. Reaching would not be as restricted as donning socks. The reacher wouldn’t be very effective in assisting with shoes and socks, and has a tendency to be overused. Stair climbing requires about 60° to 70° of hip flexion, and step height modification would not be needed. To reach to the foot, approximately 120° of hip flexion is required. If a modified procedure is chosen with the foot across the opposite thigh, at least 110° of hip flexion is still required. With only 105° of hip flexion, the patient would have most difficulty putting on socks.
Hall CM, Brody LT. Therapeutic Exercise: Moving Toward Function. Philadelphia, PA: Lippincott Williams & Wilkins; 1999: 390.

158. A physical therapist is examining a patient with an upper lumbar radiculitis. Which of the following combinations of movements is MOST likely to reproduce the patient’s pain associated with the radiculitis? 

1. Supine knee extension with hip flexion 
2. Supine knee flexion with hip flexion 
3. Prone knee flexion with hip extension 
4. Prone knee extension with hip extension 

Correct Answer: 3 
Supine knee extension with hip flexion places a stretch on the sciatic nerve, which involves lower lumbar nerve roots (L4-S3). Knee flexion with hip flexion would not place a stretch on the femoral nerve. An upper lumbar radiculitis involves the nerve roots that form the femoral nerve (L2-L4). The femoral nerve can be stretched or provoked, by extending the hip while flexing the knee. Prone hip extension would place a stretch on the femoral nerve, but there would be less stretch with the knee in extension than in flexion.
Butler DS. Mobilization of the Nervous System. 2nd ed. New York, NY: Churchill Livingstone; 1996: 136-137.

159. An adult male patient reports increasing morning stiffness and loss of spinal mobility. Radiographs of the patient’s sacroiliac joints show blurring of the joint margins. Which of the following additional findings is MOST likely to be present in this patient? 

1. Impaired sensation of the distal limbs 
2. Decreased chest-wall expansion 
3. Peripheral joint swelling 
4. Positive straight-leg raise 

Correct Answer: 2 
Peripheral neuropathies are not characteristic of ankylosing spondylitis. The symptoms and signs of morning stiffness, loss of spinal mobility, and ankylosis of the sacroiliac joints, are most suggestive of ankylosing spondylitis, a disease more frequently found among adult males. Because of the involvement of the costochondral joints, chest wall expansion is frequently restricted. Swelling of peripheral joints is not characteristic of ankylosing spondylitis. The straight leg raise test is suggestive of nerve root compression. Nerve root compression is not characteristic of ankylosing spondylitis.
Goodman CC, Boissonnault WG, eds. Pathology: Implications for the Physical Therapist. Philadelphia, PA: WB Saunders; 1998: 667-670.

160. A physical therapist is initiating intervention with a postoperative patient who is taking 20 mg of oral oxycodone (Oxycontin) for pain relief. In planning an intervention for this patient, the physical therapist should anticipate that: 

1. the patient may be hypertensive. 
2. the medication may trigger cardiac arrhythmias in the patient. 
3. the patient will have an increased likelihood for developing diarrhea. 
4. the patient may demonstrate respiratory depression. 

Correct Answer: 4 
Regarding the patient being hypertensive, the opposite is likely. Orthostatic hypotension is a potential side effect. Arrhythmias are not listed as a recognized side effect of opioids. Regarding developing diarrhea, gastrointestinal motility is decreased, so the opposite effect of constipation is a frequent problem. Opioids tend to make the medullary chemoreceptors less responsive to carbon dioxide, thus slowing down respiratory rate and inducing a relative hypoxia and hypercapnia. The respiratory response to exercise may be blunted.
Ciccone CD. Pharmacology in Rehabilitation. Philadelphia, PA: FA Davis; 1990: 155.

161. A physical therapist measures the forced expiratory volume in 1 second (FEV1) of a patient with mild pulmonary obstructive disease and determines a FEV1/FEV ratio of 60%. The therapist should recognize that the patient: 

1. has a normal FEV1 value. 
2. will require careful monitoring during aerobic exercise. 
3. should not exercise beyond 60% of the patient’s predicted maximal heart rate. 
4. should not perform aerobic exercise without physician approval. 

Correct Answer: 2 
Regarding option 1, a ratio of 75% is considered normal. Regarding option 2, the patient has a mild disease. Advanced obstructive disease can have values of 25%. Nonetheless, the patient has mild morbidity, so special precautions should be used. Regarding option 3, the patient has a mild disease and is not yet showing signs of morbidity, so special precautions should not be necessary. There would be no need to limit heart rate to 60% if the patient tolerates it. A forced vital capacity of 60% is a mild loss. Avoidance of aerobic exercise should not be required, and special physician approval related to a value of 65% should not be necessary.
Sadowsky HS. In: Hillegass EA, Sadowsky HS, eds. Essentials of Cardiopulmonary Physical Therapy. Philadelphia, PA: WB Saunders; 1994: 420.

162. A physical therapist should anticipate that an abnormal lymph node will feel: 

1. soft and non-mobile, and have an increased skin temperature. 
2. firm and non-tender, and have an increased skin temperature. 
3. firm, mobile, and tender or nontender. 
4. soft, mobile, and tender or nontender. 

Correct Answer: 3 
An abnormal lymph node should feel firm, not soft. An elevated skin temperature is not an expected accompanying feature of an abnormal lymph node. An abnormal lymph node may feel firm and nontender, but an elevated skin temperature is not an expected accompanying feature of an abnormal lymph node. An abnormal lymph node can range in feeling from firm to hard, be mobile or nonmobile, and be tender or nontender. Again, an abnormal lymph node can range in feeling from firm to hard, but would not feel soft.
BBoissonnault WG. In: Boissonnault WG, ed. Primary Care for the Physical Therapist: Examination and Triage. St. Louis, MO: Elsevier/Saunders; 2005: 110.

163. A patient with a severe cerebellar lesion can sit independently, can stand with minimal assistance, and requires moderate assistance of 1 person to walk safely. The patient desires to transfer independently from a wheelchair to a bed. Which of the following transfer techniques is MOST appropriate for this patient? 

1. Standing-pivot 
2. Manual-pivot 
3. Mechanical-lift 
4. Sliding 

Correct Answer: 4 
Standing-pivot transfer requires standing, which the patient is not able to perform safely. Manual-pivot transfer involves the assistance of another person, so would not allow for independence. Mechanical-lift transfer involves the assistance of another person, so would not allow for independence. Sliding transfer is the safest independent technique since the patient cannot stand without minimal assistance.
Humphrey F. In: Umphred DA, ed. Neurological Rehabilitation. 3rd ed. St Louis, MO: Mosby; 1995: 675.

164. A physical therapist is planning an intervention for a patient with a cerebellar lesion. The patient can maintain adequate sitting posture, but demonstrates mild difficulty with standing up from a sitting position and tends to lose balance in standing. Which of the following interventions is MOST appropriate to address this patient’s disability? 

1. Approximation through the hips and shoulders with the patient standing 
2. Rhythmical stabilization in sitting 
3. Resistive exercises to antigravity muscles 
4. Activities involving eyes open versus eyes closed in standing 

Correct Answer: 1 
Approximation reinforces stability and standing is appropriate, because the patient has the most difficulty in this position. Rhythmical stabilization would be appropriate to address a loss of balance in sitting, but this patient’s major problem is in standing balance. Resistive exercises to antigravity muscles may improve endurance for patients who are walking, but would not address the postural instability. Opening or closing the eyes has no effect on the ability to maintain standing balance in patients with cerebellar disease.
Humphrey F. In: Umphred DA, ed. Neurological Rehabilitation. 3rd ed. St Louis, MO: Mosby; 1995: 670-676.




165. A physical therapist is applying electrical stimulation to a patient, using the waveform shown in the diagram. Which of the following rationales for selecting this waveform is MOST accurate? 

1. This waveform promotes more effective pain modulation. 
2. This waveform enhances patient comfort. 
3. This waveform offers a change in frequency that promotes motor unit recruitment. 
4. This waveform offers pulse width variability that enhances motor unit recruitment. 

Correct Answer: 2 
Regarding effective pain modulation, ramps are used to obtain a more gradual activation of muscle, rather than as a method of pain modulation. Ramp controls are usually not available on stimulators designed specifically for pain control. The waveform shown is monophasic with a ramp-up and ramp-down surge of the amplitude. The gradual rise and fall in the stimulus pulse amplitude allows the gradual recruitment of nerve fibers, creating a more comfortable initiation of contraction for the patient. The declining ramp at the end of the contraction allows a smooth gradual drop in the force produced by the muscle, also enhancing comfort. The gradual onset of muscle stimulation produces contractions that more closely mimic those produced in functional activities during voluntary muscle activation. Regarding frequency and motor unit recruitment, the waveform shown has a change in amplitude, not frequency. The waveform also has a change in amplitude, not pulse width.
Alon G. In: Nelson RM, Currier DP, eds. Clinical Electrotherapy. Norwalk, CT: Appleton & Lange; 1987: 43. Robinson AJ, Snyder-Mackler L. Clinical Electrophysiology: Electrotherapy and Electrophysiologic Testing. 2nd ed. Baltimore, MD: Williams & Wilkins; 1995: 47, 159. Snyder-Mackler L, Robinson AJ. Clinical Electrophysiology: Electrotherapy and Electrophysiologic Testing. Baltimore, MD: Williams & Wilkins; 1989: 101.

166. A physical therapist is applying an electrotherapeutic modality to a patient with a 2-year history of lumbosacral dysfunction and low back pain. Using which of the following modalities and instruments is MOST appropriate for this patient? 

1. Portable stimulator delivering sensory-level stimulation 
2. Clinic-based treatment delivering motor-level stimulation 
3. Clinic-based treatment delivering sensory-level stimulation 
4. Portable stimulator delivering motor-level stimulation 

Correct Answer: 4 
Sensory level stimulation will likely be insufficient to provide pain relief, because the analgesia isn’t as long lasting as motor-level stimulation. Clinic-based treatment would limit the frequency of treatment and therefore, the overall effectiveness. The patient has a chronic problem, so will most likely require a motor-level stimulation, because it provides a longer-lasting analgesia. A portable stimulator is preferable because it will enable the treatment to continue on to a home program outside the clinic.
Snyder-Mackler L, Robinson AJ. Clinical Electrophysiology: Electrotherapy and Electrophysiologic Testing. Baltimore, MD: Williams & Wilkins; 1989: 220-221.

167. A physical therapist notes that a patient is taking 180 mg of codeine orally every 3 to 4 hours. Which of the following resulting side effects is the patient MOST likely to have? 

1. Urinary frequency 
2. Hypertension 
3. Constipation 
4. Diaphoresis 

Correct Answer: 3 
Urinary frequency is not a side effect of opioids. Hypertension is a potential side effect of stronger opioids such as morphine. Constipation is a frequent side effect of opioid analgesics, because they decrease gastrointestinal motility. Diaphoresis is a symptom of narcotic withdrawal, but is not expected in a patient who is still taking the medication.
Ciccone CD. Pharmacology in Rehabilitation. Philadelphia, PA: FA Davis; 1990: 155.

168. A physical therapist, who is newly graduated, is initiating inpatient rehabilitation with a patient who is comatose. A relative of the patient asks the physical therapist to find a more experienced therapist to work with the patient. Which of the following responses is MOST appropriate for the physical therapist to provide to the patient’s relative? 

1. “Do you believe that I am not competent to help with the rehabilitation?” 
2. “Are you concerned that I won’t be able to help your loved one recover?” 
3. “I passed my licensure examination, which indicates that I can be effective.” 
4. “Please allow me to work with this patient, and if you are still concerned, we can discuss changes.” 

Correct Answer: 2 
Option 1’s choice may create a barrier to communication, because the response is defensive. Option 2’s response indicates that the therapist is concerned with the relative’s feelings and provides the relative with the opportunity to express any concerns. Passage of the licensure examination may not convince the relative of competency and does not promote communication between the therapist and the relative. The approach of option 4 does not encourage the patient’s relative to express concerns.
Davis CM. Patient Practitioner Interaction: An Experiential Manual for Developing the Art of Health Care. 3rd ed. Thorofare, NJ: SLACK; 1998: 106.

169. While working in a private practice clinic, a physical therapist observes a patient fall in the parking lot outside the office. The patient sustains a severe laceration to the forearm. The physical therapist secures a pressure dressing to the wound site, but notes that blood is soaking through the dressing and the bandage. Which of the following actions should the therapist perform NEXT? 

1. Elevate the limb and apply pressure to the wound. 
2. Remove the dressing and bandage and start over with a tighter bandage. 
3. Apply additional dressings and bandages and apply pressure to the brachial artery. 
4. Call the patient’s physician and arrange transportation for medical care. 

Correct Answer: 1 
Elevation and pressure is the correct next step in controlling bleeding. Removal of the dressing and bandage would potentially irritate the wound and increase the hemorrhage. Elevation and pressure should be applied first, before additional dressings, bandages, and pressure to the brachial artery. While calling the patient’s physician to arrange transportation for medical care may be necessary, it does not immediately control the bleeding.
American Red Cross. First Aid/CPR/AED Program: Participant’s Booklet. Dallas, TX: American Red Cross; 2001: 17.

170. A patient comes to physical therapy with report of weakness and tingling in the lower extremities experienced in the past 2 weeks. The physical therapist suspects that this patient may have Guillain-Barré Syndrome. Which of the following questions is MOST appropriate for the physical therapist to ask to obtain evidence to support this suspicion? 

1. “Have you had a recent cold or infection?” 
2. “Have you been in an accident lately?” 
3. “Has this happened to you in the past?” 
4. “How is this affecting your work?” 

Correct Answer: 1 
Approximately 60% of Guillain-Barré Syndrome cases are associated with recent upper respiratory infections or other infections. Regarding a recent accident, traumatic causes of symptoms would help to rule out, rather than rule in the diagnosis. Guillain-Barré Syndrome is acute and does not usually recur. Therefore, this question would also help to rule out, rather than rule in the diagnosis. This question is too general and does not help to further the diagnostic suspicions.
Smith MB. In: Goodman CC, Fuller KS, Boissonault WG, eds. Pathology: Implications for the Physical Therapist. 2nd ed. WB Saunders; 2003: 1159-1160.

171. During an initial evaluation, which of the following tests is MOST appropriate to perform with a patient who has acute right-sided congestive heart failure? 

1. Sensory testing of upper extremities 
2. Pitting edema measurements in the lower extremities 
3. Resisted manual muscle testing of all extremities 
4. Reflex testing of lower extremities 

Correct Answer: 2 
While sensory testing is important in an initial exam, impaired sensation is less likely to occur in congestive heart failure by itself (as written in the question). Right-sided congestive heart failure results in dependent edema; measurements of pitting edema are appropriate to determine the severity of congestive heart failure and aid the therapist in treatment planning. Manual muscle testing is also appropriate for an initial exam. However, with acute congestive heart failure, resisted manual muscle testing is generally avoided until the congestive heart failure is more stable. Reflex testing is also important in an initial exam, but again, since this patient only has congestive heart failure, then reflexes are less likely to be impaired.
Goodman CC. In: Goodman CC, Fuller KS, Boissonault WG, eds. Pathology: Implications for the Physical Therapist. 2nd ed. WB Saunders; 2003: 411.

172. A physical therapist is preparing to evaluate a patient who had a closed reduction with cast fixation for an ankle fracture 1 day ago and is currently non-weight-bearing. Before the examination begins, the patient reports foot pain in the injured extremity. Based on the patient’s report, which of the following examinations is most appropriate to perform INITIALLY? 

1. Gait examination 
2. Transfer abilities 
3. Capillary refill in toes 
4. Blood pressure examination 

Correct Answer: 3 
Since the patient is non-weight-bearing, transfer abilities and gait examination are less likely to yield information regarding the cause of foot pain. With regard to capillary refill in toes, complications of cast fixation may include swelling and, if severe, compartment syndrome. Improperly fitted casts and/or patients leaving limbs in a dependent position can result in painful swelling that creates occlusion to distal blood flow. Testing capillary refill is an easy test to examine distal extremity perfusion. Blood pressure changes could cause changes in perfusion to distal extremities, but it would be very difficult to measure lower extremity blood pressure with the cast on.
Paz JC, West MP. Acute Care Handbook for Physical Therapists. 2nd ed. Boston, MA: Butterworth-Heinemann; 2002: 220-223, 371.

173. A patient with muscular dystrophy was weaned from mechanical ventilation 1 day ago. The patient is currently unable to independently clear secretions, despite receiving instruction in the bronchopulmonary hygiene techniques of positioning, percussion, shaking, and vibration. Which of the following interventions is MOST appropriate to help this patient clear the secretions? 

1. Nasotracheal suctioning 
2. Manual costophrenic assist 
3. Supplemental oxygen 
4. Inspiratory muscle training 

Correct Answer: 2 
While nasotracheal suctioning is a viable option to clear a patient’s secretions, it is usually the last resort when a patient does not have an artificial airway. So if option manual costophrenic assist doesn’t work, then suctioning may be needed. Any patient who has been on mechanical ventilation will likely have some respiratory muscle deconditioning. However, with superimposed neuromuscular disease, the respiratory muscle weakness will be further exacerbated. Therefore, providing manual assist at the lower ribs during cough will assist the patient in successfully clearing secretions. Supplemental oxygen can help a patient’s ventilatory muscle endurance but does not ensure that the patient will be able to generate enough force during the cough. Inspiratory muscle training would be beneficial for this patient, but not at this acute stage status, post recent ventilation wean. Once secretions are under control, then muscle training can begin.
Massery M, Cahalin LP. In: DeTurk WE, Cahalin LP. Cardiovascular and Pulmonary Physical Therapy: An Evidence-based Approach. New York, NY: McGraw-Hill Medical Publishing Division; 2004: 615-616.

174. A physical therapist is working with a patient who had a total knee arthroplasty 2 days ago. The patient’s resting electrocardiogram is shown in strip A. While gait training, the patient’s electrocardiogram changes, as shown in strip B. Based on this finding, what is the BEST action for the physical therapist to take at this time? 

1. Stop gait training and notify the nurse. 
2. Continue gait training as the heart rate is less than 100 bpm. 
3. Stop gait training and allow the patient to sit down and rest. 
4. Continue gait training, but allow the patient standing rest breaks. 


Correct Answer: 1 
The electrocardiogram change shows 3 mm ST depression, which is indicative of cardiac ischemia and an indication to stop exercise and notify medical staff. Continuing gait training would endanger the patient. Stopping and resting is a plausible option. However, the medical staff should be alerted to this situation first. Continuing gait training, despite some standing rests could allow the ischemia to progress and endanger the patient.
Hillegass EA. In: Hillegass E, Sadowsky HS, eds. Essentials of Cardiopulmonary Physical Therapy. 2nd ed. Philadelphia, PA: WB Saunders; 2001: 352.

175. A physical therapist is examining a patient who presents with unilateral lower extremity pain upon walking, which is relieved by rest. Which of the following findings reported by the patient will help the physical therapist confirm a diagnosis of intermittent claudication? 

1. Pain relief upon forward bending or sitting 
2. Pain relief upon standing 
3. Cramping pain that occurs at a predictable distance walked 
4. Numbness and tingling that occurs at a predictable distance walked 

Correct Answer: 3 
Pain relief upon forward bending or sitting and pain relief upon standing are more diagnostic for lumbar origin of pain. Regarding cramping pain that occurs at a predictable distance walked, common presentation and clinical manifestations of pain caused by vascular limitation is cramping in nature and occurs in predictable exertion levels. Numbness and tingling that occurs at a predictable distance walked are more in line with a neurologic cause of pain rather than a vascular cause.
Paz JC, West MP. Acute Care Handbook for Physical Therapists. 2nd ed. Boston, MA: Butterworth-Heinemann; 2002: 388-389.

176. A patient with a C6 spinal cord injury is progressing nicely towards achieving functional goals in an acute rehabilitation setting. However, the patient continues to report feeling winded during functional activities. The physical therapist examines the patient’s maximal inspiratory pressure and determines that the patient will benefit from inspiratory muscle training. Which of the following exercise prescriptions is MOST appropriate for this patient to begin training? 

1. 10% of maximal inspiratory pressure for 5 minutes, 1 time/day 
2. 20% of maximal inspiratory pressure for 5 to 15 minutes, 2 to 3 times/day 
3. 30% of maximal inspiratory pressure for 20 minutes, 1 time/week 
4. 50% of maximal inspiratory pressure for 5 to 15 minutes, 2 to 3 times/week 

Correct Answer: 2 
Option 1’s exercise prescription would not provide enough overloads to gain benefits. There is good evidence that inspiratory muscle training is safe and provides beneficial results for patients with respiratory muscle weakness. Option 2’s prescription was culled from the literature by Massery & Cahalin and is thus published in Cahalin’s textbook. Option 3’s prescription would not provide enough overload either. The intensity is too great to begin with, while the frequency is too little to gain benefits.
Massery M, Cahalin LP. In: DeTurk WE, Cahalin LP. Cardiovascular and Pulmonary Physical Therapy: An Evidence-based Approach. New York, NY: McGraw-Hill Medical Publishing Division; 2004: 621-623.

177. A physical therapist is prescribing therapeutic exercises for a patient to perform in the physical therapy gym. The patient had a left cerebrovascular accident 2 weeks ago with resultant right lower extremity paresis. The patient also has a history of gastroesophageal reflux disease. The physical therapist should AVOID placing the patient in which of the following positions during the exercises?

1. Sitting upright at 90° 
2. Standing 
3. Supine 
4. Sitting semi-upright at 45° 

Correct Answer: 3 
Neither the standing, sitting upright, nor semi-upright position promotes reflux. Supine position could facilitate relaxation of the lower esophageal sphincter and promote reflux from the stomach into the esophagus.
Goodman CC. In: Goodman CC, Fuller KS, Boissonault WG, eds. Pathology: Implications for the Physical Therapist. 2nd ed. WB Saunders; 2003: 634-635.

178. Which of the following postural drainage positions is MOST appropriate for a patient who has aspiration pneumonia in the right middle lobe and who had a craniotomy 2 days ago? 

1. Right sidelying with one-quarter turn towards supine 
2. Left sidelying with one-quarter turn towards supine with head of bed down approximately 20° 
3. Right sidelying with one-quarter turn towards supine with head of bed down approximately 20° 
4. Left sidelying with one-quarter turn towards supine 

Correct Answer: 4 
The traditional position to drain the right middle lobe is left sidelying, not right sidelying. Option 2 is the traditional drainage position for right middle lobe, but does not take into account the intracranial pressure issues. Option 3 is the traditional drainage position for right middle lobe, but does not take into account the intracranial pressure issues; it should be left sidelying. Again, the traditional position to drain the right middle lobe is left sidelying with one-quarter turn towards supine, head of bed down approximately 20°. However, because of the recent craniotomy, the patient’s increased intracranial pressure is a major issue to monitor and keep stable. Therefore, this traditional position should be modified to flat sidelying.
Paz JC, West MP. Acute Care Handbook for Physical Therapists. 2nd ed. Boston, MA: Butterworth-Heinemann; 2002: 694.

179. A patient who is currently participating in an outpatient cardiac rehabilitation program asks the physical therapist about intervention options for an apparent athlete’s foot fungal infection on both of the patient’s feet. The patient has been using an over-the-counter analgesic ointment for 5 days, but the condition does not seem to be improving. Which of the following actions is the correct response of the physical therapist? 

1. Refer the patient back to the cardiologist. 
2. Recommend an over-the-counter anti-fungal ointment. 
3. Tell the patient to continue using the ointment for another 5 days and then reassess. 
4. Instruct the patient to discuss the situation with a pharmacist. 

Correct Answer: 4 
The cardiologist is not the best referral, as this appears to be an integumentary problem unrelated to the cardiac condition. A physical therapist should not recommend any over the counter medications, as it is outside the scope of practice of a physical therapist. Any pharmaceutical education or advice, except “as prescribed by your physician”, is outside the physical therapist’s scope of practice. A pharmacist is the most appropriate heath care professional of those given to assist this patient.
American Physical Therapy Association. APTA Guide for Professional Conduct. Alexandria, VA: APTA; 2004: 4.1F.

180. A patient with peripheral vascular disease presents to physical therapy for evaluation and intervention. The patient used to walk for exercise, but can no longer walk to the mailbox at the end of the driveway without experiencing leg pain. This patient will MOST likely also have: 

1. relief of pain with the legs elevated. 
2. purple or brown pigmentation of the skin on the legs. 
3. relief of pain with the legs in the dependent position. 
4. a positive Homan’s sign. 

Correct Answer: 3 
Elevating the legs in the presence of arterial insufficiency decreases blood flow, which increases pain. Purple or brown pigmentation of the skin on the legs is associated with venous insufficiency, not arterial insufficiency. The patient has intermittent claudication caused by arterial insufficiency. Placing the patient’s legs in the dependent position facilitates blood flow and reduces pain. Pain with exercise indicative of intermittent claudication, not deep vein thrombosis associated with a positive Homan’s sign.
Goodman CC, Snyder TK. Differential Diagnosis in Physical Therapy. 3rd ed. Philadelphia, PA: WB Saunders; 2000: 119.

181. A physical therapist plans to use a tilt table for a patient who is having difficulty tolerating upright sitting. The therapist should stop inclining the tilt table when the patient experiences which of the following signs and symptoms? 

1. Decrease in diastolic blood pressure of 15 mm Hg 
2. Increase in systolic blood pressure of 10 mm Hg 
3. Increase in heart rate of 15 bpm 
4. Decrease in oxygen saturation to 93% 

Correct Answer: 1 
Excessive drop in blood pressure is indicative of patient intolerance to upright posture. Systolic blood pressure may increase slightly and heart rate may increase slightly. Oxygen saturation of 93% is within the acceptable range.
Pierson FM, Fairchild SL. Principles and Techniques of Patient Care. 3rd ed. Philadelphia, PA: WB Saunders; 2002: 216.

182. A patient involved in a motor vehicle accident 1 week ago has multiple fractures of the lower extremities and is non-weight-bearing bilaterally. The patient lives with an elderly parent and is scheduled for discharge soon. Which of the following transfers is MOST appropriate in this situation? 

1. Standing-pivot 
2. 1-person lift 
3. Sliding-board 
4. 2-person lift 

Correct Answer: 3 
Since the patient is non-weight-bearing bilaterally, the standing-pivot transfer is contraindicated. Since the caregiver is elderly, neither a 1-person nor a 2-person lift, is a safe choice. Since the caregiver is elderly and the patient is non-weight-bearing bilaterally, the sliding-board transfer is the best choice of the options given.
Pierson FM, Fairchild SL. Principles and Techniques of Patient Care. 3rd ed. Philadelphia, PA: WB Saunders; 2002: 146.

183. A physical therapist is evaluating a patient who has a chest tube. The therapist accidentally knocks over the collection reservoir. The therapist should return the reservoir to the upright position and: 

1. hang it on an intravenous pole. 
2. place it on the patient’s bedside table. 
3. hang it from the side of the bed. 
4. contact the physician. 

Correct Answer: 3 
The reservoir must be placed below the level of the heart. Notifying the nurse would be sufficient. Notifying the physician is unnecessary.
Paz JC, West MP. Acute Care Handbook for Physical Therapists. 2nd ed. Boston, MA: Butterworth-Heinemann; 2002: 796-797.

184. A 22 year-old patient is hospitalized awaiting a lung transplant due to cystic fibrosis. The patient’s physician is interested in an objective measure of the patient’s preoperative endurance. Which of the following tests is MOST appropriate for the physical therapist to administer to this patient? 

1. VO2 max treadmill test 
2. 2-step test 
3. Submaximal exercise test on a cycle ergometer 
4. 6-minute walk test 

Correct Answer: 4 
The patient is too ill for either the VO2 max treadmill test or the 2-step test. The submaximal exercise test is not as sensitive or specific a test as the 6-minute walk test. The 6-minute walk test is designed for acutely ill cardiopulmonary patients.
Cahalin LP. In: DeTurk WE, Cahalin LP. Cardiovascular and Pulmonary Physical Therapy: An Evidence-based Approach. New York, NY: McGraw-Hill Medical Publishing Division; 2004: 256-261.

185. A patient is limited in shoulder abduction, as shown in the displayed radiograph. Which of the following mobilization techniques is MOST likely to assist this patient in achieving increased shoulder abduction? 

1. Dorsal glide 
2. Ventral glide 
3. Caudal glide 
4. Cranial glide 


Correct Answer: 3 
Neither dorsal glide, ventral glide, nor cranial glide, would improve abduction. The radiograph shows limitation of glenohumeral motion causing the reduction in shoulder abduction. Caudal glide assists with improving abduction.
Edmond SL. Manipulation and Mobilization: Extremity and Spinal Techniques. St. Louis, MO: Mosby; 1993: 24-34.

186. A 55 year-old patient is referred to physical therapy from the emergency room following placement of a long-leg cast. The patient sustained a right femur fracture, when hit by a car while bike riding. The patient is nauseous, but has good balance during initial attempts at non-weight-bearing gait training on the right lower extremity. The patient lives alone in an apartment on the second floor. Which of the following is the MOST appropriate assistive device for this patient? 

1. Standard walker 
2. Axillary crutches 
3. Quad cane 
4. 2 straight canes 

Correct Answer: 2 
A standard walker on stairs is not as safe as crutches. As the patient was previously active, has stairs at home, and must be non-weight-bearing on one lower extremity, axillary crutches would provide the greatest safety margin and independence. A quad cane would not be sufficient, since the patient is non-weight-bearing on one lower extremity. The patient would not be able to be non-weight-bearing on 1 lower extremity with 2 straight canes.
Minor MD, Minor SD. Patient Care Skills. 3rd ed. Norwalk, CT: Appleton & Lange; 1995: 287.

187. A patient who sustained a right cerebrovascular accident presents with a flaccid arm. During muscle testing, the patient is able to shrug the left shoulder. The MOST accurate explanation for shoulder movement is that the right cerebrovascular accident: 

1. has affected the right shoulder and not the left shoulder. 
2. did not affect the vagus nerve (X), which innervates the upper trapezius muscle. 
3. did not affect the spinal accessory nerve (XI), which innervates the upper trapezius muscle. 
4. has affected the left biceps and triceps muscles, but not the deltoid muscles. 

Correct Answer: 3 
A right cerebrovascular accident affects the left shoulder, not the right shoulder. The upper trapezius is controlled by spinal accessory nerve (XI), not the vagus nerve (X). The spinal accessory nerve (XI) (supplied by the corticobulbar tract) was apparently not affected by the stroke and accounts for the patient’s ability to shrug a flaccid arm. The deltoid does not shrug the shoulder. 
Paz JC, West MP. Acute Care Handbook for Physical Therapists. 2nd ed. Boston, MA: Butterworth-Heinemann; 2002: 291-292.

188. A physical therapist is evaluating the cranial nerves of a child who has a medulloblastoma. The child’s right eye deviates medially. This child has impairment of which of the following cranial nerves? 

1. Oculomotor (III) 
2. Trochlear (IV) 
3. Abducens (VI) 
4. Vagus (X) 

Correct Answer: 3 
The oculomotor nerve (III) controls the inferiomedial eye muscles. The trochlear nerve (IV) controls inferiolateral eye movement. The abducens nerve (VI) controls lateral eye movement. Damage to this nerve causes the eyeball to deviate medially due to weakness of the lateral rectus muscle. The vagus nerve (X) does not control the eye muscles.
Goetz CG, Pappert EJ, eds. Textbook of Clinical Neurology. Philadelphia, PA: WB Saunders; 1999: 943-944.

189. Following a cerebrovascular accident, a patient is evaluated for cognitive and perceptual dysfunctions. The patient is asked to stack several wooden blocks. After picking up a block, the patient is unable to determine how the block should be used. This dysfunction is MOST likely due to: 

1. homonymous hemianopsia. 
2. astereognosis. 
3. unilateral neglect. 
4. constructional apraxia. 

Correct Answer: 4 
Homonymous hemianopsia describes a visual impairment. There is no evidence of visual limitations. Astereognosis is the inability to recognize an object by handling the object without looking at the object. Unilateral neglect describes the inability to register and integrate stimuli from one side of the body. Constructional apraxia describes a cognitive dysfunction in which a patient does not know what to do with the blocks.
Unsworth C. In: O’Sullivan SB, Schmitz TJ, eds. Physical Rehabilitation: Assessment and Treatment. 4th ed. Philadelphia, PA: FA Davis; 2001: 967-972; 988-990.

190. During the physical therapy examination of a patient with shoulder pain, the patient’s active and passive shoulder flexion is 110° with an abrupt, hard end feel at the end of the available range. This patient MOST likely has: 

1. subacromial bursitis. 
2. rotator cuff tear. 
3. adhesive capsulitis. 
4. bicipital tendinitis. 

Correct Answer: 3 
Regarding option 1, commonly, passive and active ranges of motion are not equally limited and are more likely to be problematic for shoulder abduction. An empty end feel is more likely. Regarding option 2, range of motion is more limited secondary to muscle weakness. Also, a hard end feel would not be present. Examination findings of shoulder flexion limitation equal in both passive and active range of motion and accompanied by a hard end feel are indicative of adhesive capsulitis. Commonly passive range of motion is not limited and does not have a hard end feel present.
Magee DJ. Orthopedic Physical Assessment. 4th ed. Philadelphia, PA: Saunders; 2002: 27.

191. A patient has a history of diabetes, hypertension, and chronic heart failure. The patient’s venous filling time is less than 15 seconds, Homan’s sign is negative, and rubor dependency test is not slowed. These findings support which of the following diagnoses? 

1. Lymphedema 
2. Arterial insufficiency 
3. Intermittent claudication 
4. Chronic venous insufficiency 

Correct Answer: 4 
Unresolved edema is not indicated and would be more prevalent in lymphedema. A venous filling time of less than 15 seconds is suggestive of arterial insufficiency. Intermittent claudication is related to activity, not to dependency tests. A venous filling time of less than 15 seconds is suggestive of venous insufficiency. 
(try to check, there’s a conflict here - - its supposed to be- - venous filling time of more than 15sec is arterial insufficiency while venous filling time of less than 15sec is venous insufficiency)
Knight CA. In: O’Sullivan SB, Schmitz TJ, eds. Physical Rehabilitation: Assessment and Treatment. 4th ed. Philadelphia, PA: FA Davis; 2001: 586-587, 594.

192. A patient with a right transtibial prosthesis describes right-knee buckling in the foot flat (loading response) stance phase of gait. The presence of which of the following conditions in the patient is MOST likely the cause of this problem? 

1. Excessive plantarflexion 
2. Stiff heel cushion 
3. Low shoe heel 
4. Excessive foot inset 

Correct Answer: 2 
Excessive prosthetic plantarflexion can cause insufficient knee flexion. A stiff heel cushion can cause excessive knee flexion and therefore buckling. A low prosthetic shoe heel can cause delayed knee flexion. Excessive prosthetic foot inset can cause excessive lateral thrust.
Edelstein JE. In: O’Sullivan SB, Schmitz TJ, eds. Physical Rehabilitation: Assessment and Treatment. 4th ed. Philadelphia, PA: FA Davis; 2001: 665. Table 20-1.

193. A patient was recently diagnosed with fibromyalgia. Which of the following is the MOST appropriate intervention plan for general strengthening for this patient? 

1. Low resistance, low repetitions 
2. Low resistance, high repetitions 
3. High resistance, low repetitions 
4. High resistance, high repetitions 

Correct Answer: 1 
Patients with fibromyalgia are not able to tolerate strengthening programs that involve either high repetitions or high resistance; therefore, only low resistance and low repetitions would be potentially beneficial for this patient population.
Hall CM, Brody LT. Therapeutic Exercise: Moving Toward Function. Philadelphia, PA: Lippincott Williams & Wilkins; 1999: 203-205.

194. A patient presents to physical therapy with the diagnosis of a medial meniscus tear of the right knee. Which of the following signs and symptoms MOST strongly supports this diagnosis? 

1. Mechanical locking 
2. Decreased pain with weight-bearing 
3. Posterior knee swelling 
4. Atrophy of hamstrings 

Correct Answer: 1 
A history of mechanical locking is a common symptom of knee medial meniscus tear. Pain is commonly increased with weight-bearing, not decreased weight-bearing. Swelling would more likely be evident anteriorly, not posteriorly. Quadriceps atrophy is more likely, not hamstrings atrophy.
Hertling D, Kessler RM. Management of Common Musculoskeletal Disorders. 3rd ed. Philadelphia, PA: Lippincott; 1996: 352-353.

195. A patient with a cerebellar stroke has received functional balance training for 4 weeks. Which of the following tests is MOST appropriate to measure the effectiveness of the physical therapy intervention? 

1. Romberg Test 
2. Berg Balance Scale 
3. Fugl-Meyer Assessment 
4. Barthel Index 

Correct Answer: 2 
Although the Romberg Test is a measure of the role of vision in balance, it alone would not be the most appropriate measure for functional balance. The Berg Balance Scale is an objective measure of static and dynamic balance abilities and consists of 14 commonly performed functional tasks; therefore, it is the most appropriate tool to use as a measure of intervention effectiveness. The Fugl-Meyer Assessment is appropriate for cortical strokes and would not be the most appropriate for a cerebellar stroke. It is not as comprehensive in balance tasks as the Berg Balance Scale. The Barthel Index is a more global instrument and is not as focused on functional balance as the Berg Balance Scale.
Guccione AA. In: O’Sullivan SB, Schmitz TJ, eds. Physical Rehabilitation: Assessment and Treatment. 4th ed. Philadelphia, PA: FA Davis; 2001: 316. O’Sullivan SB. In: O’Sullivan SB, Schmitz TJ, eds. Physical Rehabilitation: Assessment and Treatment. 4th ed. Philadelphia, PA: FA Davis; 2001: 193-197.

196. A physical therapy student’s learning style is described as a preference for active experimentation and concrete experience. For mastering evaluation of musculoskeletal dysfunction, which of the following educational strategies would be the LEAST effective approach for the supervising physical therapist to use with this student? 

1. Practicing and providing feedback on examination skills 
2. Role-playing the interview process 
3. Assigning readings about the dysfunction 
4. Self-pacing performance of the evaluation 

Correct Answer: 3 
Neither practice nor feedback of examination skills, role playing the interview process, nor self-faced performance, involves active learning with other people in everyday situations. Readings do not meet the learning needs of active experimentation nor concrete experience. Each of the other options involves active learning with other people in everyday situations. Active learning relies on feeling and intuition rather than logic and reasoning.
Shepard K, Jensen G, eds. Handbook of Teaching for Physical Therapists. 2nd ed. Newton, MA: Butterworth-Heinemann; 2002: 59.

197. A patient is receiving physical therapy intervention for rheumatoid arthritis, which is in remission. Which of the following interventions is MOST appropriate for this patient? 

1. Contract-relax stretching for tight structures 
2. End-range mobilization techniques 
3. Elimination of functional activities of involved parts 
4. Strengthening exercises for weak muscles 

Correct Answer: 4 
Contract-relax stretching for tight structures are not recommended for soft tissues compromised by rheumatoid arthritis. Grade IV mobilization techniques are contraindicated for soft tissues compromised by rheumatoid arthritis. The elimination of functional activities is not required. Strengthening exercises are the most appropriate intervention for a patient with rheumatoid arthritis in remission.
Hall CM, Brody LT. Therapeutic Exercise: Moving Toward Function. Philadelphia, PA: Lippincott Williams & Wilkins; 1999: 190.

198. During gait evaluation, a physical therapist notes that a patient demonstrates a shorter, left step length and excessive left knee flexion during the left midstance phase. Which of the following problems is MOST likely the cause of this gait dysfunction? 

1. Left hamstrings contracture 
2. Right iliopsoas weakness 
3. Left hip flexion contracture 
4. Right quadriceps weakness 

Correct Answer: 1 
Left hamstrings contracture is a fixed, mechanical limitation. It is the most likely cause of the gait impairment, as it directly affects both the knee joint during the midstance phase and the step length. It is the only option that can contribute to both of the gait impairments. Neither right iliopsoas weakness, left hip flexion contracture weakness, nor right quadriceps weakness, contributes to both gait impairments.
Neumann DA. Kinesiology of the Musculoskeletal System: Foundations for Physical Rehabilitation. St. Louis, MO: Mosby; 2002: 530-535.

199. A patient is being evaluated for possible carpal tunnel syndrome and a nerve conduction velocity test is performed. Which of the following findings would MOST strongly support the diagnosis? 

1. Abnormal nerve conduction velocity at the elbow 
2. Decreased latency at the carpal tunnel 
3. Increased latency at the carpal tunnel 
4. Abnormal nerve conduction velocity at the forearm 

Correct Answer: 3 
Nerve conduction above and below the local nerve compression is usually normal. Latency is typically increased, not decreased, across the carpal tunnel compression site. Nerve conduction above and below the local nerve compression is usually normal.
Portney LG, Roy SH. In: O’Sullivan SB, Schmitz TJ, eds. Physical Rehabilitation: Assessment and Treatment. 4th ed. Philadelphia, PA: FA Davis; 2001: 233-235.

200. A therapist is examining muscle strength in a patient. The patient is asked to move the leg along the path indicated by the arrow and to hold the leg there while the therapist applies resistance to the leg. The patient moves through the range of motion shown in the photograph and is able to take maximal resistance. This intervention should address which of the following problems? 

1. Tightness of the hip internal rotators 
2. Weakness of the gluteus minimus and medius 
3. Correction of the right lateral trunk shift 
4. Piriformis weakness 

Correct Answer: 1 
The fact that the patient has normal muscle strength of the hip external rotators along with limitation of range of motion indicates either tightness of the internal rotators or hip capsule tightness. The gluteus medius and minimus are internal rotators of the hip. The trunk shift is a normal substitution pattern found in individuals that are lacking hip range of motion. The problem is related to the hip, not the trunk. The patient’s piriformis shows normal strength, so it is not weak.
Reese NB. Muscle and Sensory Testing. Philadelphia, PA: WB Saunders; 1999: 281.

3 comments:

  1. Very good study material . More similar materials please

    ReplyDelete

  2. Please don't read and just ignore it because it's really happening to me and my Dad all thanks to Dr Aboda ❣️ .
    I don't really know how to Express or say thank you for the love and help you have shown me and my family. This is to inform everyone out there with different kinds of health challenges, as I know there are still a lot of people suffering from different kinds of health situations and they are looking for solutions. This is a wonderful testimony, I don't really believe it's going to happen so fast. God bless 💕 Dr Aboda for curing my dad with his marvelous remedies. My Dad has been stuffing from ischemic stroke disease for the past 3 year now and we have use different kinds of treatment prescribed by family doctor and so many other hospitals doctor's but still yet there was no good results or healing instead, on till a friend of mine introduce me to Dr Aboda healing treatment, I really want to say a big thanks to Dr Aboda for healing my Dad from ischemic stroke . My advice to anyone with any kind of disease or virus please contact Dr Aboda for a cure.
    Via Email: drabodasolution@gmail.com

    Contact: +2349012479806

    https://www.facebook.com/Drabodasolutionhome

    For more information about Dr Aboda you can contact me vivianjefferson958@gmail.com

    ReplyDelete
  3. Wynn Slots for Android and iOS - Wooricasinos
    A free app for slot machines from WRI Holdings 토토사이트 Limited that 출장샵 lets wooricasinos.info you play the septcasino popular games, such as free video slots, table games and https://septcasino.com/review/merit-casino/ live casino

    ReplyDelete